#9 Rosh Review

¡Supera tus tareas y exámenes ahora con Quizwiz!

Question: What is the most common type of hernia in adults?

Answer: Inguinal hernia, which represents 75% of all hernias. Rapid Review Strangulated Hernia Vascular compromise of hernia contents Usually incarcerated prior to strangulation ABX Surgical emergency

Question: If infected with hepatitis B virus, what is the risk of developing hepatocellular carcinoma?

Answer: 20-45%, more if the virus was parenterally transmitted. Rapid Review Acute Hepatitis HAV: fecal-oral, shellfish, alone (no carrier), asymptomatic, acute HBV: HBsAg: active infection Anti-HBs: recovered or immunized Anti-HBc IgM: early marker of infection, positive in window period Anti-HBc IgG: best marker for prior HBV HBeAg: high infectivity Anti-HBeAb: low infectivity HCV: IVDA, chronic, cirrhosis, carcinoma, carrier HDV: dependent on HBV coinfection HEV: fecal-oral (enteric) high mortality rate among pregnant (expectant) patients, epidemics, HAV and HEV are fecal-oral: "The vowels hit your bowels" Autoimmune hepatitis: young females Alcoholic hepatitis: moderate transaminase elevation, AST>ALT Supportive rx

Question: At what age should patients be referred to a gynecologist to rule out Bartholin gland cancer?

Answer: 40.

Question: What is the desired heart rate in the treatment of chronic stable angina with beta-blocker therapy?

Answer: 55-60/min.

Question: Name two steroid-sparing alternative medications used for pulmonary sarcoidosis.

Answer: Azathioprine and methotrexate. Rapid Review Sarcoidosis African-Americans, females Primary target organ: lungs Parotid enlargement Hypercalcemia CXR: bilateral hilar adenopathy Biopsy: noncaseating granulomas Steroids

Question: What is an example of a drug that reduces cardiac contractility?

Answer: Beta blockers.

Question: What medication is indicated in the treatment of infant botulism?

Answer: Botulism Immune Globulin Intravenous (Human) (BIG-IV) has been shown to reduce hospital length of stay and time on mechanical ventilation. Rapid Review Botulism Toxicity: Infantile, foodborne, and wound types > 70% of cases are infantile > 20% linked to honey exposure, not ingestion Toxin secretion causes illness by acting on pre-synaptic nerves to block the release of acetylcholine Symptoms: bulbar palsy hypotonia respiratory failure Diagnosis made with stool culture and toxin assay, no blood tests available Antibiotics are contraindicated

Question: Which smoking cessation medication is recommended for patients who are concerned about post-cessation weight gain?

Answer: Bupropion.

Question: What antibiotic is best known for causing aplastic anemia?

Answer: Chloramphenicol. Rapid Review Pancytopenia Aplastic anemia Chloramphenicol Leishmania donovani Megaloblastic anemia Paroxysmal nocturnal hemoglobinuria (PNH) Radiation sickness Transfusion-associated GVHD

Question: What infectious condition should all PROM patients be evaluated for?

Answer: Chorioamnionitis, an inflammation of the fetal membranes (amnion and chorion) due to a bacterial infection. Rapid Review Premature Rupture of Membranes (PROM) PROM: membrane rupture prior to labor Preterm PROM (PPROM): PROM occurring < 37 weeks Fluid ferning + blue nitrazine paper = amniotic fluid Admission, OB consultation

Question: Antisocial personality disorder is part of which cluster group?

Answer: Cluster B.

Question: What medical condition is associated with Streptococcus bovis endocarditis?

Answer: Colorectal cancer. Rapid Review Endocarditis Patient will be complaining of fever, rash, cough and myalgias PE will show Fever, Roth spots, Osler nodes, Murmur, Janeway lesions, Anemia, Nailbed hemorrhages, Emboli (FROM JANE) Diagnosis is made by echocardiography and Duke's criteria Most commonly caused by: IVDA: S. aureus, tricuspid Native valve: Streptococci, mitral Treatment is antibiotics Comments: GI malignancy: S. bovis

Question: What are considered confirmatory tests of group A streptococcal infection when evaluating a child with possible rheumatic fever?

Answer: Confirmatory testing may be either a positive throat culture, positive rapid antigen test, or rising streptococcal antibody titer (ASO). Rapid Review Rheumatic Fever: Sequela from Group A streptococcal infection Diagnosis made with 1 major + 2 minor Jones criteria OR 2 major Jones criteria Major criteria carditis polyarthritis chorea erythema marginatum subcutaneous nodules Minor criteria arthralgias fever elevated ESR and CRP

Question: Which medical treatment is known to be beneficial in patients with croup by decreasing edema in the laryngeal mucosa?

Answer: Corticosteroids. Rapid Review Chlamydia Pneumoniae Patient will be a between two and nineteen weeks of life With a history of congestion Complaining of cough, hoarseness and malaise PE will show staccato cough and a lack of fever Most commonly caused by Chlamydia pneumoniae Treatment is macrolide antibiotic

Question: What activity has been shown to have the greatest effect in preventing or delaying the onset of diabetes type 2?

Answer: Dietary modifications and increased physical activity.

Question: Which two calcium channel blockers should be used most cautiously in conjunction with beta-blockers due to the increased risk of AV conduction depression?

Answer: Diltiazem and verapamil, the two nondihydropyridine calcium channel blockers, pose the greatest risk of depressing AV node conduction and sinus node automaticity when combined with beta-blockers.

Question: What procedure is used to treat paraphimosis when minimally invasive techniques have failed?

Answer: Dorsal slit procedure.

Question: What antibiotics can be used for prophylaxis in penicillin allergic patients?

Answer: Doxycycline, moxifloxacin, or clindamycin plus ciprofloxacin. Rapid Review Cat Bite Most commonly caused by Pasteurella multocida Treatment is irrigate, leave wound open, amoxicillin-clavulanate Complications: Osteomyelitis, Tenosynovitis

Question: What is the antibiotic of choice for psittacosis in children and pregnant women?

Answer: Erythromycin. Rapid Review Psittacosis Patient with a history of exposure to birds Complaining of high fevers, severe headache, myalgias, nonproductive cough PE will show hepatosplenomegaly CXR will show patchy perihilar or lower lobe infiltrates Most commonly caused by Chlamydia psittaci Treatment is doxycycline

Question: Proteinuria and nephrotic syndrome lead to what kinds of casts seen on microscopic examination of urine?

Answer: Fatty casts. Rapid Review Urinalysis: Causes of False Negatives Protein: dilute, acidic Leukocytes: abx, glycosuria, proteinuria Nitrite: ascorbate Bilirubin: ascorbate, aged sample, rifampicin, UV light exposure Glucose: ascorbate

Question: As compared to Dupuytren's contracture, where is the pathology located in stenosing tenosynovitis (trigger finger)?

Answer: Fibrotic nodules occurring in the digital flexor tendons. Rapid Review Mallet Finger Patient will be a basketball or volleyball player With a history of forced hyperflexion of the DIP Complaining of inability to extend DIP Treatment is volar splinting DIP in extension Comments: If untreated -> swan neck deformity

Question: What laboratory test can aid in the diagnosis of menopause in a woman who is status post hysterectomy?

Answer: Follicle-stimulating hormone (usually > 70 IU/L). Rapid Review Menopause Decreased ovarian follicles --> decreased estrogen Average age: 51 Menopause before 40: premature ovarian failure Increased FSH is specific HAVOCS: Hot flashes, Atrophy of Vagina, Osteoporosis, CAD, Sleep abnormalities Vaginal bleeding: r/o malignancy

Question: In the setting of liver cirrhosis, why is glucagon unlikely to help with episodes of hypoglycemia?

Answer: Glucagon acts by increasing the breakdown of glycogen to glucose. Patients with liver cirrhosis do not have adequate glycogen stores for glucagon to be effective. Rapid Review Hepatic Encephalopathy Toxin accumulation due to liver dysfunction Causes: infection, dehydration, medications, GI bleed, hypoxia, hypoglycemia ↑ Ammonia CSF glutamine correlates with severity Classification system: Grade 1: mild cognitive dysfunction, irritability Grade 2: lethargy, disoriented, confusion asterixis Grade 3: somnolence inability to follow commands Grade 4: coma Rx: lactulose

Question: At what lead level do you start chelation therapy?

Answer: Greater than or equal to 45. Rapid Review Lead Poisoning: Patient will be a child Complaining of headache, joint pain, and constipation X-ray will show hyperdense lines at metaphyses (lead lines) Labs will show microcytic, hypochromic anemia, and basophilic stippling on peripheral smear Treatment is oral succimer or IV EDTA (Calcium disodium edetate)

Question: What in-office physical exam technique may aid in triggering an absence seizure in susceptible patients?

Answer: Guided hyperventilation. Rapid Review Absence Seizures (petit mal) Patient will be 5 - 10 y/o Complaining of sudden mental status alteration without motor activity, blank stare EEG would show symmetric 3-Hz spike and wave activity Treatment is ethosuxamide Comments: no aura or postictal state

Question: Name some other causes of parotitis?

Answer: HIV, tuberculosis, Sjogren's syndrome and sarcoidosis. Rapid Review Mumps Viral paramyxovirus Winter/spring Parotitis Orchitis Deafness Aseptic meningitis

Question: What is the test to order as screening for iron deficiency anemia as recommended by the American Academy of Pediatrics?

Answer: Hemoglobin.

Question: What are some extracolonic findings associated with ulcerative colitis?

Answer: Hepatitis and cirrhosis, uveitis and episcleritis, and erythema nodosum and pyoderma gangrenosum. Rapid Review Ulcerative Colitis Bloody diarrhea, crampy abdominal pain Continuous mucosal inflammation Always involves the rectum Crypt abscesses Toxic megacolon Colon cancer Sulfasalazine, 5-ASA derivative

Question: What is the traditional treatment for sprains?

Answer: Ice, elevation, and analgesia with NSAIDs. Immobilization is used to provide protection and comfort in the initial management. Rapid Review Ankle Sprain Partial or complete tearing of ≥ 1 ligaments MC due to inversion mechanism MC sprained: anterior talofibular ligament (ATFL) Grade I: microscopic tear, grade II: incomplete tear, grade III: complete tear RICE

Question: What is the recommended medication regimen for outpatient treatment of PID?

Answer: Intramuscular ceftriaxone plus PO doxycycline and PO metronidazole. Rapid Review Pelvic Inflammatory Disease (PID) Patient will be a Female With a history of multiple sexual partners or unprotected sex or both Complaining of lower abdominal pain, cervical motion tenderness, painful sexual intercourse PE will show mucopurulent cervical discharge, "Chandelier sign" Most commonly caused by Chlaymidia Treatment is ceftriaxone + doxycycline Comments: Fitz-Hugh-Curtis syndrome: perihepatitis + PID

Question: What is an alternative regimen to treat endometritis?

Answer: Intravenous ceftriaxone and Intravenous metronidazole. Rapid Review Endometritis Patient will be a woman 2 - 3 days post c-section Complaining of fever, abdominal pain, foul smelling lochia PE will show purulent vaginal discharge, CMT, uterine tenderness Labs will show leukocytosis Most common postpartum infection Treatment is: Post cesarean section: Clindamycin + gentamicin, post vaginal delivery: Ampicillin + gentamicin

Question: What specific gastrointestinal disorder is associated with Henoch-Schönlein purpura?

Answer: Intussusception. Rapid Review Henoch-Schonlein Purpura (HSP) Patient will be 4-12 years old With a history of recent URI Complaining of abdominal pain, arthralgia, and a rash that began on buttocks and lower extremities PE will show maculopapular rash (palpable purpura) that is non-pruritic Most commonly caused by IgA mediated vasculitis Treatment is supportive care Comments: Complications include nephropathy, intussusception

Question: How does iron ingestion lead to metabolic acidosis?

Answer: Iron, at toxic levels, poisons the cellular mitochondria leading to lactic acidosis. Rapid Review Metabolic Acidosis pH <7.35 + HCO3- <24 ​Normal AG: Hyperalimentation, Addison Disease, RTA, Diarrhea, Acetazolamide, Spironolactone, Saline infusion (HARDASS) ↑ AG: Methanol, Uremia, DKA, Propylene glycol, Iron or INH, Lactic acidosis, Ethylene glycol, Salicylates (MUDPILES) Tachypnea Winter's formula: PCO2 = 1.5(HCO3-) + 8 +/- 2 ​PCO2 < Winter's formula calculation = respiratory alkalosis PCO2 > Winter's formula calculation = respiratory acidosis

Question: What is the leading cause of acquired heart disease in kids younger than 5 years of age?

Answer: Kawasaki disease. Rapid Review Kawasaki Disease Patient will be a child < 4 years old With a history of high fever for 5 days Complaining of conjunctivitis, rash, adenopathy, strawberry tongue, hand/feet edema, fever Treatment is IVIG + aspirin Comments: #1 cause of pediatric acquired heart disease, risk for coronary artery aneurysm Mnemonic: CRASH and burn: Conjunctivitis, Rash, Adenopathy, Strawberry tongue, Hand/feet edema, Fever

Question: Is the zoster vaccine a live attenuated or inactivated vaccine?

Answer: Live attenuated.

Question: What imaging test is most helpful in the workup of vertebrobasilar insufficiency?

Answer: MRI of the brain. Rapid Review Peripheral Vertigo CN VIII, vestibular apparatus Onset: sudden Hearing loss, tinnitus Positional Nystagmus: uni-directional, never vertical, fatigable, can be inhibited No neurologic sx

Question: Paired, narrow, white, transverse bands that disappear when pressure is applied to the nail bed are called what?

Answer: Muehrcke's lines.

Question: Zollinger-Ellison syndrome can be part of what autosomal dominant familial syndrome?

Answer: Multiple Endocrine Neoplasia type 1. Rapid Review Mallory-Weiss Syndrome Patient will have a history of drinking alcohol and forceful vomiting Complaining of hematemesis Diagnosis is made by upper endoscopy Caused by an incomplete tear in the esophagus mucosa

Question: Does natural infection with diphtheria induce immunity?

Answer: No. Rapid Review Diphtheria Patient will be complaining of URI-like illness, sore throat, low fever PE will show bull neck, cervical adenopathy, "pseudomembrane" on pharynx and "grains of salt" on tonsils Most commonly caused by corynebacterium diphtheria (anaerobic Gram positive)

Question: What histopathological biopsy finding is typically seen in patients with sarcoidosis?

Answer: Noncaseating granulomas.

Question: What is the most common radiographic finding for a Salter-Harris Type V fracture?

Answer: Normal. Rapid Review Salter-Harris Fractures I: S (Slipped epiphysis) II: A (fracture Above physis), most common III: L (fracture beLow physis) IV: T (fracture Through physis) V: R (wRecked physis) I/II rx: nonoperative IV/V rx: surgery required Negative radiographs do not r/o a Salter I fracture

Question: What structure is commonly compressed and affects vision in those with a pituitary adenoma?

Answer: Optic chiasm. Rapid Review Pituitary Tumors Visual sx (bitemporal hemianopia) Prolactinoma: (most common), amenorrhea, impotence Growth hormone tumor: gigantism (children), acromegaly (adults) ACTH secreting TSH secreting Nonfunctioning

Question: What is the name of the tender nodules found on the fingertips of patients with infective endocarditis?

Answer: Osler nodes. Rapid Review Endocarditis Patient will be complaining of fever, rash, cough and myalgias PE will show Fever, Roth spots, Osler nodes, Murmur, Janeway lesions, Anemia, Nailbed hemorrhages, Emboli (FROM JANE) Diagnosis is made by echocardiography and Duke's criteria Most commonly caused by: IVDA: S. aureus, tricuspid Native valve: Streptococci, mitral Treatment is antibiotics Comments: GI malignancy: S. bovis

Question: What is the other name for exophthalmos?

Answer: Proptosis, although this term is usually reserved for non-endocrine causes of globe protrusion, while exophthalmos is specifically used for globe protrusion due to an underlying endocrinopathy. Rapid Review Dacryoadenitis Lacrimal gland enlargement Unilateral Severe pain, swelling Supratemporal region

Question: Name 5 urea-splitting bacteria?

Answer: Proteus, Pseudomonas, Klebsiella, Staphylococcus, and Mycoplasma. Rapid Review Nephrolithiasis Patient will be complaining of flank pain radiating to groin PE will show a patient that won't lay still and hematuria Diagnosis is made by helical CT Most commonly caused by calcium oxalate Struvite: staghorn calculi, urease producing bacteria Uric acid: radiolUcent on xray, gout Cystine: children with metabolic diseases Most common location is the ureterovesiclular junction (UVJ) Treatment is: < 5 mm: likely to pass spontaneously > 8 mm: unlikely to pass, lithotripsy

Question: What is the most common gram-negative pathogen identified in neutropenic fever?

Answer: Pseudomonas aeruginosa. Rapid Review Neutropenic Fever > 38.3ºC + ANC < 500 Obtain cultures Rx: admission, ABX

Question: What are the four cardiac anatomical abnormalities in tetralogy of Fallot?

Answer: Pulmonary stenosis, overriding aorta, ventricular septal defect, and right ventricular hypertrophy. Rapid Review Ventricular Septal Defect PE will show a loud, harsh, holosystolic murmur at the lower left sternal border Diagnosis is made by echo Treatment is most close spontaneously Comments: most common pathologic murmur in childhood

Question: What does the HBeAg serologic marker for hepatitis B indicate?

Answer: Replication and infectivity. Its presence represents high levels of DNA in the serum and higher rates of transmission.

Question: In addition to gabapentin, what are some medications useful in treating the vasomotor symptoms of menopause?

Answer: Selective serotonin reuptake inhibitors and clonidine have some utility in managing vasomotor symptoms during menopause. Rapid Review Atrophic Vaginitis Patient will be a postmenopausal woman Complaining of dyspareunia, dryness, bleeding, itching PE will show pale, dry, shiny epithelium Most commonly caused by decrease in estrogen Treatment is lubricants, moisturizers, topical estrogen (2nd line)

Question: What is Seidel's test?

Answer: Streaming of aqueous humor leaking from the anterior chamber during fluorescein examination secondary to penetration of the cornea. Rapid Review Corneal Abrasion Patient will be complaining of pain, photophobia, tearing, or foreign body sensation Diagnosis is made by fluorescein stain Treatment: Most are self-limiting, antipseudomonal for contact lens wearers

Question: End of life care is an example of which type of prevention?

Answer: Tertiary prevention.

Question: Into which anatomic regions can a patient hemorrhage enough to become hypotensive due to blood loss?

Answer: The chest, abdomen, retroperitoneum, and thighs.

Question: The ulnar nerve is responsible for the sensation of what part of the hand?

Answer: The palmar and dorsal surface of the 5th digit and the medial aspect of the 4th digit.

Question: What is the classic echocardiographic finding in constrictive pericarditis?

Answer: Thickened pericaridum. Rapid Review Constrictive Pericarditis Fibrotic changes leading to reduced diastolic filling No wall enlargement Pericardial knock on cardiac auscultation Kussmaul's sign and pulsus paradoxus Pericardiectomy

Question: What is the most common location for zoster to occur in?

Answer: Thorax, followed by the face (trigeminal nerve). Rapid Review Herpes Zoster Age, immunodeficiency Reactivation of latent VZV in dorsal root ganglion Grouped vesicles on an erythematous base, dermatomal Postherpetic neuralgia, Ramsay Hunt syndrome, zoster ophthalmicus Acyclovir

Question: Besides Tinea capitis, which other tinea infection should be treated with systemic antifungal medication?

Answer: Tinea unguium (onychomycosis).

Question: What type of collagen is affected in Osteogenesis Imperfecta?

Answer: Type 1 Collagen.

Question: Metformin is first-line treatment in which type of diabetes mellitus?

Answer: Type 2.

Question: What non-pharmacologic therapy for pediculosis is available?

Answer: Wet combing.

Question: Ventricular tachycardia is most difficult to distinguish from which other dysrhythmia?

Answer: Wide-complex supraventricular tachycardia (SVT with aberrancy such as a bundle-branch block). Rapid Review Ventricular Tachycardia AV dissociation, wide-complex Rate typically 150-200 >3 consecutive ectopic ventricular beats Monomorphic, polymorphic Bidirectional: digoxin toxicity Pulseless: immediate defibrillation Unstable: synchronized cardioversion Stable: procainamide, amiodarone, synchronized cardioversion (refractory) If unsure, manage all wide complex tachycardias as ventricular tachycardia Ventricular Fibrillation Wide-complex, disorganized No cardiac output or pulse Incompatible with life Defibrillation, ACLS Atrial Flutter Sawtooth pattern Atrial rate: 250-300/minute Ventricular rate: 150- +/-30 AV node conducts every 2 or 3 atrial impulses Atrial Fibrillation Irregularly irregular No P waves Narrow QRS unless conduction block or accessory pathway Variable ventricular response rate

Which of the following is most consistent with a complication that most commonly occurs 6 to 12 weeks after acute hepatitis? WBC decreased, RBC decreased, platelets decreased WBC decreased, RBC increased, platelets increased WBC increased, RBC decreased, platelets decreased WBC increased, RBC increased, platelets increased

Correct Answer ( A ) Explanation: Aplastic anemia is a known complication of acute hepatitis. This affects up to 2% of all patients after their initial illness. Hepatitis-associated aplastic anemia most often affects adolescent boys and young men, is most commonly seen 6-12 weeks after hepatitis, and can be fatal if untreated. Its etiology is unclear. There is no known association with blood transfusions, drugs, or toxins. Most patients have been seronegative for hepatitis A, B, and C, but it is believed to be the result of autoimmune bone marrow failure. Lab findings show a decrease in the WBC, RBC, and platelet counts. Although one might expect the WBC count or platelet count (B, C, and D) to rise, as acute phase reactants, marrow failure precludes this in aplastic anemia.

A 4-week-old boy presents with a 2-week history of increasing dyspnea, cough, and poor feeding. On examination you note conjunctivitis, and a chest examination reveals tachypnea and rales. A chest X-ray shows hyperinflation and diffuse interstitial infiltrates. Which of the following is the most likely etiologic agent? Chlamydia trachomatis Parainfluenza virus Respiratory syncytial virus Staphylococcus species

Correct Answer ( A ) Explanation: Chlamydia trachomatis infections in newborns can manifest as pneumonia and conjunctivitis. Chlamydial pneumonia is usually seen in infants 3-16 weeks of age, and they frequently have been sick for several weeks. Fewer than 10% of neonates born to women with active chlamydia infection during labor develop pneumonia. The infant appears nontoxic and is afebrile, but is tachypneic with a prominent staccato cough. Physical examination reveals diffuse rales with few wheezes. The chest film shows hyperinflation and diffuse interstitial or patchy infiltrates and blood work frequently reveals eosinophilia. In addition to chest radiographs and blood work, specimens should be collected from the nasopharynx. Erythromycin, 50 mg/kg divided into 4 doses for 14 days, is the treatment of choice. Conjunctivitis is present in about 50% of cases. Symptoms include conjunctival injection, various degrees of ocular discharge, and swollen eyelids. This infection is transmitted vaginally from an infected mother, and can present within the first 15 days of life. The diagnostic standard is to culture a conjunctival swab from an everted eyelid. Treatment for conjunctivitis is the same as for pneumonia. Prophylaxis with silver nitrate solution or antibiotic ointments does not prevent vertical perinatal transmission of C. trachomatis, but it will prevent ocular gonococcal infection and should therefore be administered. Staphylococcal pneumonia (D) has an abrupt onset. The infant appears very ill and has a fever. Signs of tachypnea, dyspnea, and localized or diffuse bronchopneumonia or lobar disease may be present with prominent leukocytosis on blood work. Respiratory syncytial infections (C) usually cause lower respiratory tract infections, such as bronchiolitis. These children have cough, fever, rhinorrhea, wheezing, labored respirations and occasionally hypoxia. Chlamydial infections may be differentiated from respiratory syncytial infections by a history of conjunctivitis and a subacute onset. Patients with parainfluenza virus (B) typically present with a history of coryza and low-grade fever; they then develop the classic barking cough associated with croup.

Which of the following combinations of hepatitis B serologic markers is indicative of hepatitis B immunity secondary to vaccination? HBsAg negative, anti-HBc negative, anti-HBs positive HBsAg negative, anti-HBc positive, anti-HBs positive HBsAg positive, anti-HBc positive, IgM anti-HBc negative, anti-HBs negative HBsAg positive, anti-HBc positive, IgM anti-HBc positive, anti-HBs negative

Correct Answer ( A ) Explanation: HBsAg negative, anti-HBc negative, anti-HBs positive is the serologic marker combination indicative of hepatitis B immunity due to administration of the hepatitis B vaccination. HBsAg is a marker for the hepatitis B surface antigen, a protein on the surface of the hepatitis B virus. Presence of this protein in the body conveys that the person is currently infected and that they have either acute or chronic hepatitis B. Anti-HBc is the total hepatitis B core antibody that appears at the onset of symptoms and remains for life. This marker indicates a previous or ongoing infection with hepatitis B. Anti-HBs is the hepatitis B surface antibody, which is positive when a patient has recovered from the disease or when a patient has been immunized against the disease either by vaccination or prior infection. When a patient has immunity due to vaccination, the HBsAg will be negative since they are not acutely or chronically infected, the anti-HBc will be negative as they have not had a previous or an active infection, and the anti-HBs will be positive because they were immunized against the virus by a vaccination. HBsAg negative, anti-HBc positive, anti-HBs positive (B) is indicative of immunity to hepatitis B virus because of natural infection. HBsAg negative indicates that the person is not acutely infected with hepatitis B, anti-HBc positive indicates that they were at some point infected with the virus, and anti-HBs positive indicates that they have recovered from the disease. HBsAg positive, anti-HBc positive, IgM anti-HBc negative, anti-HBs negative (C) demonstrates that the patient is chronically infected with hepatitis B. HBsAg positive indicates that the patient is infected, whether it's acutely or chronically. Anti-HBc positive is a marker that remains for life and will always be positive if a patient was at one point infected. IgM anti-HBc demonstrates the acuity of the infection and would be positive if a patient was infected within the past six months. In the case of chronic infection, the IgM anti-HBc is negative. Anti-HBs is negative if a patient has not yet recovered from the disease. HBsAg positive, anti-HBc positive, IgM anti-HBc positive, anti-HBs negative (D) indicates that a person is acutely infected with the hepatitis B virus. HBsAg is positive when a patient is currently infected. Anti-HBc is positive at the onset of symptoms of hepatitis B, IgM anti-HBc is positive when a patient has been infected for fewer than six months and indicates the acuity of the disease, and anti-HBs is negative if the patient has not yet recovered from the infection.

A 15 year old boy presents with acute onset, atraumatic eye swelling. Examination reveals superolateral orbital edema and tenderness. Which of the following is the most likely diagnosis? Dacryoadenitis Dacryocystitis Exopthalmos Ptosis

Correct Answer ( A ) Explanation: Inflammation of the lacrimal glands is called dacryoadenitis. It is most commonly due to viral or bacterial infection. Unilateral, superolateral orbital pain and swelling predominate. There may be preauricular lymphadenopathy, discharge and excessive tearing. The differential includes orbital cellulitis, chalazion, hordeolum, exophthalmos, ptosis and lacrimal tumor. Any discharge should be cultured. CT evaluation is considered if tumor is suspected. Acute cases are treated with warm compresses, NSAIDs and cephalosporins if underlying bacterial infection is suspected. Chronic cases should involve evaluation of underlying disorders such as thyroid disease and sarcoidosis. Dacryocystitis (B) is an infectious obstruction of the nasolacrimal duct. onset is usually rapid with a clinical presentation of unilateral, severe pain, redness and epiphora (overflow of tears). This condition occurs in the inframedial region of the orbit. Exopthalmos (C), commonly seen in Grave's disease, is anterior bulging of the eyeball out of the orbit (globe protrusion). Ptosis (D) is drooping or falling of the upper eyelid.

Which of the following is a minor rather than a major Jones criterion in the diagnosis for acute rheumatic fever? Fever New murmur Polyarthritis Subcutaneous nodules

Correct Answer ( A ) Explanation: Rheumatic fever is a known delayed sequela of infection with group A Streptococcal infection. Fever is a minor criterion for acute rheumatic heart disease. The Jones criteria, created in 1944 and updated in 1992, are used to diagnose acute rheumatic fever (ARF). Rheumatic heart disease is an important yet uncommon sequela of rheumatic fever. Acute rheumatic fever incidence is declining in the U.S. and other developed countries and reported to be fewer than 10,000 cases per 100,000 individuals. Diagnosis of ARF is possible using one major plus two minor Jones criteria or two major Jones criteria. Major criteria include carditis (new murmur), polyarthritis, chorea, erythema marginatum (non-pruritic, erythematous rings, found on trunk and inner limbs), and subcutaneous nodules (painless collagen collections on back of wrists, elbows, and knees). Minor criteria include arthralgias, fever, elevated acute phase reactants (erythrocyte sedimentation rate and C-reactive protein). New murmur (B) represents carditis, or valvulitis, which is a major Jones criterion. Polyarthritis (C) and subcutaneous nodules (D) are also incorrect because they are also major Jones criteria.

A 70-year-old woman with a long history of coronary artery disease is seen in cardiology clinic for routine follow up. She complains of continued angina despite medical therapy. She reports chest pain every time she walks to her mailbox. It does not occur at rest and is relieved by sublingual nitroglycerin. Her medications include carvedilol, amlodipine, daytime transdermal nitroglycerine, sublingual nitroglycerin, aspirin, and simvastatin. On physical exam her BP is 105/72, HR is 51 and RR 16. What is the next step in management? Add ranolazine Increase the dose of carvedilol Increase the dose of simvastatin Schedule 24 hour usage of transdermal nitroglycerin

Correct Answer ( A ) Explanation: The addition of ranolazine should be considered in patients with chronic stable angina who remain symptomatic despite optimal doses of beta blockers, calcium channel blockers and nitrates. Stable angina refers to chest discomfort that occurs predictably and reproducibly at certain levels of exertion and is relieved by rest or nitroglycerine. This patient remains symptomatic on carvedilol, amlodipine and two different preparations of nitroglycerine. Ranolazine is a novel anti-anginal agent that causes selective inhibition of the late sodium channel. It is approved for the treatment for chronic stable angina for those who have failed standard medical therapy. It has been shown to be effective in reducing anginal symptoms and improving exercise capacity when added to conventional medical therapy. Ranolazine can prolong the QT interval and it should be used with caution in patients with kidney or liver disease. It is not safe to increase the dose of carvedilol (B) as this patient's blood pressure and pulse are too low for up-titration of her beta-blocker. Statin therapy is important for the treatment of this patient's coronary artery disease; however it is not appropriate to increase the dose of simvastatin (C) in this situation as statin therapy does not have anti-anginal properties. Long acting nitroglycerin preparations are very useful in the treatment of chronic stable angina; however, they require a nitrate free period of 8-12 hours, typically at night, to obviate nitrate tolerance. Therefore, 24 hour use of transdermal nitroglycerin (D) would not be beneficial.

What is the most common cause of vertigo? Benign paroxysmal positional vertigo Labyrinthitis Meniere's disease Vertebrobasilar insufficiency

Correct Answer ( A ) Explanation: The majority of vertigo is caused by peripheral diseases and are not life-threatening. Benign paroxysmal positional vertigo (BPPV) is the most common of the peripheral disorders. Vertigo is defined as an illusion of motion. It is classically described as sensation of the room spinning. There are a number of causes of vertigo. The critical action in determining the cause is to separate peripheral vertigo from central vertigo. Peripheral vertigo can be caused by vestibular neuritis, labyrinthtits, Meniere's disease, acute otitis media, perilymphatic fistula, trauma to the labyrinth, acoustic neuroma and BPPV. Each of these etiologies have classic findings and symptoms that can aid in differentiating the causes. BPPV is typically caused by the presence of an otolith in the labyrinth system. The Dix-Hallpike maneuver can be used to support the diagnosis of BPPV. After performing a Dix-Hallpike, an Epley maneuver can be performed to expel the otolith from the labyrinth. Labyrinthitis (B) is typically preceded by an ear, nose and throat infection and is accompanied by hearing loss. Meniere's disease (C) typically has concurrent tinnitus with vertigo. Vertebrobasilar insufficiency (D) is a central cause of vertigo that may precede an ischemic CVA.

A 34-year-old type I diabetic presents for an evaluation of her diabetes. She is on basal and bolus insulin, taking multiple daily insulin injections. Upon reviewing her glucose levels, you notice a trend of an elevated pre-breakfast glucose level and dinnertime levels averaging 90 mg/dL. She also reports a 3 am glucose of 60 mg/dL. What is the most appropriate change to make in her regimen? Decrease the evening dose of basal insulin Decrease the evening dose of bolus insulin Increase the evening dose of basal insulin Increase the evening dose of bolus insulin

Correct Answer ( A ) Explanation: The patient's early morning hyperglycemia is due to the Somogyi effect. This is a rebound hyperglycemia (pre-breakfast) that is a response to hypoglycemia (nighttime/early morning). Although the phenomenon is controversial, it is thought to occur when a patient becomes hypoglycemic (due to too much nighttime basal insulin) which causes an adrenergic outpouring (epinephrine, growth hormone, cortisol, and glucagon) that activates gluconeogenesis and glycogenolysis that ultimately leads to a reactive hyperglycemia. Treatment will be to decrease the evening dose of long-acting insulin or give bedtime snacks. Increasing the evening dose of basal insulin (C) would be the treatment if the patient were experiencing the Dawn phenomenon. The Dawn phenomenon is secondary to the surge of growth hormone which antagonizes insulin action leading to elevated pre-breakfast glucose levels. By checking the blood glucose level at 3 am, one can differentiate between the Somogyi effect (low glucose at 3 am) and Dawn phenomenon (high glucose level). Changing the evening dose of bolus insulin (B and D) will have minimal effect on pre-breakfast insulin levels due to the short acting nature of the insulin.

A 54-year-old man presents complaining of epigastric pain that started several hours ago. The pain is moderate and sharp but does not radiate. He has a history of hypertension and exploratory laparotomy 20 years ago. His vital signs on presentation are T 37°C, HR 95, and BP 136/80. His exam reveals a firm, mildly tender protruding mass in the epigastric region with no overlying skin changes. Which of the following is the next best step? Apply gentle steady pressure to the mass Obtain computed tomography scan of the abdomen Perform bedside abdominal aortic ultrasound Perform incision and drainage of the mass

Correct Answer ( A ) Explanation: This patient is presenting with a ventral hernia through the incision site from his exploratory laparotomy. Incisional hernias account for up to 20% of all abdominal wall hernias. They are often the result of excess wall tension or inadequate wound healing. Risk factors for the development of incisional hernias include obesity, age, wound infection, and certain medical conditions that increase the intraabdominal pressure. All hernias fall into one of three categories: reducible, incarcerated, or strangulated. Reducible hernias are soft and easy to replace through the hernia defect. Incarcerated hernias are firm, often painful, and non-reducible by direct manual pressure. Strangulation occurs as a consequence of incarceration and results in impaired blood flow, leading to ischemia, necrosis, and obstruction. Skin changes overlying the hernia site may be seen, and patients are often toxic in appearance. A strangulated hernia is a true surgical emergency. Abdominal aortic aneurysm (C) should be considered in this patient. However, this condition generally presents as a pulsatile mass in patients with chronic hypertension. An incision and drainage (D) is appropriate for an abscess. This mass is firm, whereas an abscess is fluctuant. To reduce the potential for ischemia and necrosis, there should be no delay at reduction (B).

Which one of the following Tinea infections in children always requires systemic antifungal therapy? Tinea capitis Tinea corporis Tinea cruris Tinea pedis

Correct Answer ( A ) Explanation: Tinea infections are caused by dermatophytes and are classified by the involved site. The most common infections in prepubertal children are tinea corporis and tinea capitis, whereas adolescents and adults are more likely to develop tinea cruris, tinea pedis, and tinea unguium (onychomycosis). In the United States, tinea capitis (scalp) most commonly affects children of African heritage between three and nine years of age. Early disease can be limited to itching and scaling, but the more classic presentation involves scaly patches of alopecia with hairs broken at the skin line and crusting. Tinea capitis may progress to kerion, which is characterized by boggy tender plaques and pustules. Tinea capitis must be treated with systemic antifungal agents because topical agents do not penetrate the hair shaft. However, adjunct treatment with selenium sulfide shampoo or 2% ketoconazole shampoo should be used for the first two weeks because it may reduce transmission. For many years, the first-line treatment for Tinea capitis was griseofulvin because it has a long track record of safety and effectiveness. However, randomized clinical trials have confirmed that newer agents, such as terbinafine and fluconazole, have equal effectiveness and safety and shorter treatment course. Tinea corporis, Tinea cruris, and Tinea pedis are generally responsive to topical creams such as terbinafine and butenafine, but oral antifungal agents may be indicated for extensive disease, failed topical treatment or immunocompromised patients. Tinea cruris (C), also known as jock itch, most commonly affects adolescent and young adult males, and involves the portion of the upper thigh. The scrotum itself is usually spared in Tinea cruris, but involved in candidiasis. Tinea corporis (B), also known as ringworm, typically presents as a red, annular, scaly, pruritic patch with central clearing and an active border. Lesions may be single or multiple and the size generally ranges from 1 to 5 cm, but larger lesions and confluence of lesions can also occur. Tinea corporis may be mistaken for many other skin disorders, especially eczema, psoriasis, and seborrheic dermatitis. A potassium hydroxide (KOH) preparation is often helpful when the diagnosis is uncertain based on history and visual inspection. Tinea pedis (D), athletes foot, typically involves the skin between the toes, but can spread to the sole, sides, and dorsum of the involved foot .The acute form presents with erythema and maceration between the toes, sometimes accompanied by painful vesicles.

In which of the following patients is a zoster vaccination indicated? A 26-year-old pregnant woman A 61-year-old man who recalls having chicken pox as a child A 7-year-old boy with asthma A 75-year-old woman with chronic lymphocytic leukemia

Correct Answer ( B ) Explanation: A zoster vaccine is indicated in a 61-year-old man who recalls having chicken pox as a child. The CDC recommends the zoster vaccine for use in people 60-years-old and older to prevent shingles. This is a one-time vaccination. Anyone 60-years of age or older should get the shingles vaccine, regardless of whether they recall having had chickenpox or not. Varicella-zoster virus infection causes two clinically distinct forms of disease. Primary infection with varicella-zoster virus results in varicella, characterized by vesicular lesions in different stages of development on the face, trunk and extremities. Herpes zoster is related to reactivation of varicella-zoster virus infection and is characterized by a painful, unilateral vesicular eruption in a restricted dermatomal distribution. Individuals who have not had varicella and are exposed to a patient with herpes zoster are at risk for developing primary varicella and not herpes zoster. Vaccination reduces the risk of zoster and postherpetic neuralgia. A 7-year-old boy with asthma (C) would receive the varicella vaccine, not the zoster vaccine. A zoster vaccine would not be recommended in a 26-year-old pregnant woman (A) or a 75-year-old woman with chronic lymphocytic leukemia (D) as pregnancy and patients who are highly immunocompromised are contraindications to receiving this vaccination.

A 66-year-old man presents after a landscaping injury. He was pruning bushes with motorized shears when he slipped and cut his right thigh. He put a towel on the wound and was able to walk to the Emergency Department. He has no other injuries and his vital signs are within normal limits. He takes warfarin for atrial fibrillation. When you examine his wound, you note a 7 cm simple laceration with blood continuously oozing from the defect. Which of the following is the appropriate next step? Apply a tourniquet proximal to the wound Apply direct pressure to the wound Irrigate the wound Reverse his anticoagulation

Correct Answer ( B ) Explanation: Bleeding is a leading cause of mortality in patients suffering from trauma. Rapid identification and control of external hemorrhage is part of the primary trauma survey. Even with normal vital signs, a significant volume of blood can be lost. Control of the bleeding can prevent further blood loss. Arterial bleeding is under pressure and will spurt, while venous bleeding will generally be a continuous slow ooze. Direct pressure to the wound is an effective technique to gain early control of a bleeding injury. Pressure compresses the vasculature and gives time for a clot to form at the site of the injury. Applying a tourniquet proximal to the wound (B) is appropriate when a patient is exsanguinating from an extremity wound. In this case, the patient has active slow bleeding and direct pressure is an appropriate, less aggressive first step in bleeding control. Irrigating the wound (C) is an important step in preventing infection and promoting wound recovery, however it is important to control the bleeding first. If the bleeding continues, reversal of his anticoagulation (D) may be necessary. However, reversal is generally not necessary to control a relatively isolated, small wound.

Which of the following conditions is associated with a pericardial knock on auscultation? Acute pericarditis Constrictive pericarditis Pericardial tamponade Restrictive cardiomyopathy

Correct Answer ( B ) Explanation: Constrictive pericarditis is associated with a pericardial knock. On cardiac auscultation, an early diastolic sound, the pericardial knock, may be heard at the apex 60 to 120 msec after the second heart sound (S2). The pericardial knock sounds like a ventricular gallop but occurs earlier than the S3 of heart failure, which it may mimic. The knock is due to accelerated right ventricular inflow in early diastole and early myocardial distention, followed by an abrupt slowing of further ventricular expansion. There is usually no pericardial friction rub. Constrictive pericarditis is pathologically distinct from acute pericarditis. Constrictive pericarditis is caused by the resultant inflammatory and reparative process from a pericardial injury. This leads to a fibrous thickening of the pericardium. Clinical presentation mimics heart failure and restrictive cardiomyopathy. Physical exam is associated with Kussmaul's sign (inspiratory neck vein distention) and pulsus paradoxus. Severe cases require surgical treatment with a pericardiectomy. Acute pericarditis (A) is associated with a pericardial friction rub. It is best heard with the diaphragm of the stethoscope at the lower left sternal border or apex when the patient is sitting and leaning forward or in the hands-and-knees position. The classic cardiac auscultation findings of pericardial tamponade (C) include distant heart or soft heart sounds. Restrictive cardiomyopathies (D) result from systemic disorders such as amyloidosis, sarcoidosis, hemochromotosis, scleroderma, carcinoid heart disease, and endomyocardial fibrosis. Findings on physical exam depend on the stage or severity of myocardial involvement. An S3 is almost always present, and an S4 is often heard.

A 7-year-old boy presents with his mom because his teacher says he "zones out" all day at school. His mom denies hyperactivity or destructive, disobedient actions but says he often "stares off" during meals for a few seconds. An EEG shows a generalized 3-Hz spike and wave pattern. Which of these interventions is first-line for his treatment? Atomoxetine (Strattera®) Ethosuximide (Zarontin®) Gabapentin (Neurontin®) Methylphenidate (Ritalin®)

Correct Answer ( B ) Explanation: Ethosuximide (Zarontin®) is the first-line medication for treating absence seizures as it is most likely to reduce frequency of epileptic spells without causing adverse effects on a child's attentiveness. Absence seizures typically present in childhood as frequent, sudden, brief periods of unconsciousness, with or without automatic movements. An EEG is critical to the diagnosis of absence seizures as it will demonstrate a classic 3 Hz generalized spike-and-wave pattern. Absence seizure are unique in their comparatively abrupt ending and lack of a post-ictal period. Atomoxetine (Strattera®) (A) is a non-stimulant medication that is considered second-line in the treatment of attention deficit hyperactivity disorder (ADHD). Gabapentin (Neurontin®) (C) has low efficacy in managing generalized seizures, though may be used for benign partial seizures in children. Methylphenidate (Ritalin®) (D) is a stimulant medication that is considered first-line in the treatment of attention deficit hyperactivity disorder (ADHD).

A 12-year-old uncircumcised boy presents to the emergency department with a complaint of inability to retract his foreskin to its normal position. While he was showering, he retracted his foreskin to clean his penis. He called his mother for help when he could not get his foreskin back into place. Which of the following is the most likely diagnosis? Balanitis Paraphimosis Phimosis Priapism

Correct Answer ( B ) Explanation: Paraphimosis occurs in uncircumcised males when entrapment of the foreskin behind the glans penis causes venous and lymphatic congestion, resulting in inability to retract the foreskin to its normal position. Risk factors include genitourinary procedures, sexual activity, penile trauma, and males who forget to replace the foreskin after cleaning or urination. Paraphimosis typically causes significant pain and is a true urologic emergency. Treatment includes the timely reduction of the foreskin back over the glans penis. Urology consultation should be ordered for any signs of arterial compromise of the glans penis. Balanitis (A) is an inflammation of the glans penis often caused by poor hygiene. Phimosis (C) occurs when the foreskin cannot be retracted over the glans penis. Priapism (D) is a prolonged erection of the penis.

Which of the following laboratory results signifies acute hepatitis? HBsAg +, anti-HBc IgG +, HBeAg -, HBV DNA + HBsAg +, anti-HBc IgM +, HBeAg +, HBV DNA + HBsAg -, anti-HBs +, HBeAg -, HBV DNA - HBsAg -, anti-HBs -, HBeAg -, HBV DNA -

Correct Answer ( B ) Explanation: Hepatitis has many etiologies, including hepatitis A, B, C, D and E viruses, as well as cytomegalovirus, varicella-zoster virus, herpes-simplex virus and Epstein-Barr virus. There are other nonviral causes, including nonalcoholic fatty liver disease, alcoholic hepatitis, ischemic hepatitis, acetaminophen hepatotoxicity and autoimmune hepatitis. Hepatitis B (HB) is transmitted via blood. Acute presentations are 70% subclinical, 30% as jaundice with elevated serum aminotransaminases, and <1% as fulminant hepatitis. Diagnosis is made via several serologic and virologic tests. HB surface antigen (HBsAg) appears before symptoms occur. Anti-HB surface antigen antibody (anti-HBs) defines resolution of acute infection and immunity. HB E antigen (HBeAg) marks the period of viral replication and increased infectivity, while anti-HBe antibody signifies the end of infection and decreased infectivity. Anti-HB core antibody (anti-HBc IgM) is the first antibody to appear and indicates acute infection. Anti-HBc antibody (anti-HBc IgG) indicates a previous or ongoing infection. HB viral DNA (HBV DNA) presence signifies acute replication in the liver. If a patient looks like they have acute hepatitis, but anti-HBc IgG is found in conjunction with no HBeAg (A), then it is a chronic, and not acute, hepatitis. HBsAg -, anti-HBs +, HBeAg -, HBV DNA - (C) denotes immunization. HBsAg -, anti-HBs -, HBeAg -, HBV DNA - (D) occurs in a patient without current or past infection with, nor immunization against, hepatitis B virus.

A 4-year-old boy presents with a 5-day history of fever. On exam he is noted to have non-tender anterior cervical lymphadenopathy, non-exudative conjunctival injection, and swollen hands and feet. Which of the following is the most common complication of this condition? Aplastic anemia Coronary artery aneurysm Glomerulonephritis Severe thrombocytopenia

Correct Answer ( B ) Explanation: Kawasaki disease (KD), also known as mucocutaneous lymph node syndrome, is one of the most common vasculitides in children. It typically presents in children younger than 4 years, most commonly around age 2 and has a male predominance. Clinical manifestations are characterized by fever of at least five days duration. In addition, there must be at least four of the following: erythema of the lips and oral mucosa, conjunctivitis, cervical lymphadenopathy, and edema or erythema of the extremities. The disease is self-limited, however a complication of Kawasaki disease is coronary artery aneurysm. Therefore, when the diagnosis is suspected, prompt treatment is critical. A cardiac echocardiogram should be performed at the time of diagnosis, 2-3 weeks later, and 6-8 weeks after the onset of illness. KD is treated initially with IVIG and high dose aspirin followed by maintenance aspirin. Aplastic anemia (A) is a known complication of several infections such as human parvovirus B19 or HIV, however it is not associated with Kawasaki disease. Glomerulonephritis (C) is a sequela of streptococcal pharyngitis. Kawasaki disease is associated with thrombocytosis by the fifth day of the illness, not thrombocytopenia (D).

Which of the following is characteristic of Mallory-Weiss syndrome? Gastrin secreting tumor of the pancreas Lacerations of the gastric cardia due to forceful vomiting Noniatrogenic traumatic injury to esophagus Spontaneous esophageal hematoma

Correct Answer ( B ) Explanation: Mallory-Weiss syndrome is characterized by a laceration of the gastric cardia due to forceful vomiting. The laceration is felt to result from shearing forces on the gastroesophageal junction and proximal stomach as it herniates through the diaphragm from high intra-abdominal pressures due to forceful vomiting. In accordance with Laplace's law, this shearing force has its greatest effect when there is a hiatal hernia, thus exposing a relatively large volume dilated sac to high wall tension. It is not surprising that the majority of patients who sustain a Mallory-Weiss tear have a hiatal hernia. Although most tears will occur within 2 cm of the gastroesophageal junction, the likelihood of a more distal tear in the proximal portion of the stomach is increased when a larger hiatal hernia is present. Any action that results in an abrupt increase in intra-abdominal pressure increases the chance of a Mallory-Weiss tear. Such actions include forceful coughing, straining, retching during endoscopy, transesophageal echocardiography, and cardiopulmonary resuscitation. Other factors that predispose to tearing include alcohol and aspirin use. Zollinger-Ellison Syndrome (A) is caused by a non-beta islet cell gastrin secreting tumor of the pancreas that stimulates the acid-secreting cells of the stomach to maximal capacity. Noniatrogenic traumatic injury to the esophagus (C) may occur through either penetrating or, less commonly, blunt injuries. Blunt trauma resulting in esophageal perforation is exceedingly rare; most cases have occurred in the cervical esophagus after motor vehicle accidents from the steering wheel or seat belt. Spontaneous esophageal hematoma (D) is a rare entity in which an abrupt bleed occurs between the mucosa and muscularis propria of the esophageal wall, often for a long length of the esophagus. The term spontaneous is somewhat of a misnomer in the sense that several underlying factors have been identified that may predispose to hematoma formation.

Nitrate therapy works by which of the following mechanisms? Reducing afterload Reducing both preload and afterload Reducing cardiac contractility Reducing preload

Correct Answer ( B ) Explanation: Nitrates work by reducing both preload and afterload. Nitroglycerine was the first treatment for angina pectoris and dates back to the 1800s. It still remains first-line drug therapy for many patients. Nitrates dilate veins and coronary arteries and to a lesser extent systemic arteries by relaxing vascular smooth muscle. Thus, nitrates reduce preload by increasing venous capacitance and improve coronary blood flow by coronary vasodilatation. Decreased preload lowers left ventricular end-diastolic pressure, thereby decreasing wall stress, resulting in a decrease in myocardial oxygen demand. At higher doses the afterload effects cause a drop in systemic blood pressure further decreasing wall stress and oxygen demand. Nitrates do not have a direct effect on cardiac chronotropy or inotropy. The primary adverse effects induced by nitrate therapy include hypotension, headache, and tachycardia. Nitrates should be avoided in patients with one or more of the following: systolic blood pressure less than 90 mm Hg, heart rate less than 50/min, or heart rate greater than 100/min. It should also be avoided in known or suspected right ventricular infarction, in patients who have taken a phosphodiesterase inhibitor for erectile dysfunction within the last 24 hours, in patients with hypertrophic cardiomyopathy or severe aortic stenosis. Nitrates do not work by only reducing afterload (A) or preload (D). Nitrates do not directly reduce cardiac contractility (C).

Which of the following disorders causes a normal anion gap metabolic acidosis? Cyanide exposure Diabetic ketoacidosis Diarrhea Salicylate ingestion

Correct Answer ( C ) Explanation: Diarrhea is a common cause of normal anion gap metabolic acidosis. Metabolic acidosis is defined as a reduced serum bicarbonate concentration. Normal anion gap metabolic acidosis is thought to be less immediately dangerous than anion gap metabolic acidosis. Normal anion gap metabolic acidosis can be caused by a variety of conditions including rapid infusion of 0.9% saline, renal tubular acidosis, ingestion of acetazolamide and calcium chloride and hypoaldosteronism. Cyanide (A), diabetic ketoacidosis (B) and salicylate ingestion (D) are all causes of increased anion gap metabolic acidosis.

A 58-year-old woman with no significant medical history presents for an annual wellness physical. She says it has been 16 months since she had any menstrual bleeding and notes moderate vaginal discomfort that makes coitus difficult for her. Which of the following recommendations would be most appropriate for this patient? Avoidance of vaginal intercourse Prescribe a vaginal ring containing estradiol Prescribe an oral conjugated estrogen preparation Prescribe gabapentin taken nightly

Correct Answer ( B ) Explanation: Prescribing a vaginal ring that contains 2 mg of estradiol to be placed once every 3 months is an appropriate initial intervention for patients with symptoms of post-menopausal vaginal atrophy. A daily intravaginal estradiol tablet is also effective. Vaginal atrophy is a very common problem in post-menopausal women that occurs due to thinning of the vaginal mucosa in the setting of decreased estrogen secretion. Additionally, low estrogen levels may impair the production of natural vaginal lubrication leading to dyspareunia. A pelvic exam in post-menopausal women often shows a pale, smooth vagina with a decrease in size of the cervix and ovaries. In addition to vaginal atrophy, post-menopausal women may complain of vasomotor symptoms, or "hot flushes." Systemic estrogen or estrogen/progestin preparations usually help with this problem. When selecting a hormone replacement, clinicians should recall that women with an intact uterus should never receive a systemic, unopposed estrogen due to the risk of endometrial hyperplasia and carcinoma. Post-menopausal women should also be counseled on their increased risk of osteoporosis as estrogen levels drop. A daily intake of 1200 mg of elemental calcium and 800 international units of vitamin D should be initiated. A daily weight-bearing exercise program will also be helpful. Avoidance of vaginal intercourse (A) may actually be harmful to patients with vaginal atrophy, as continuing to engage in intercourse usually helps prevent further tissue shrinkage. Recommending use of over-the-counter lubricants can aid patients in continuing coitus. Gabapentin taken nightly (D) will not benefit women with complaints of vaginal atrophy. This treatment would be more effective if the patient's post-menopausal symptoms manifested primarily as hot flushes. Oral conjugated estrogen preparations (C) are an inappropriate choice to manage this patient's symptoms due to the risks associated with using systemic, unopposed estrogen. In women with an intact uterus, estrogen should always be prescribed with an opposing progestin regimen to prevent endometrial hyperplasia and consequently endometrial carcinoma.

A 36-year-old veterinarian presents with myalgias, dry cough, and severe headache. His vital signs include blood pressure 138/74 mm Hg, heart rate 82 beats/minute, temperature 39°C, and oxygen saturation 94% on room air. He has hepatosplenomegaly on abdominal exam. His chest X-ray shows patchy perihilar infiltrates. What of the following is the most appropriate antibiotic for this patient? Amoxicillin-clavulanate Doxycycline Levofloxacin Trimethoprim-sulfamethoxazole

Correct Answer ( B ) Explanation: Psittacosis is caused by Chlamydia psittaci, an obligate intracellular gram-negative organism. It is harbored in avian species making bird owners, veterinarians, and pet-shop employees particularly susceptible to infection. Patients present with high fevers, severe headache, myalgias, nonproductive cough, and hepatosplenomegaly. Chest X-rays show patchy perihilar or lower lobe infiltrates. Patients may have proteinuria and elevated liver transaminases. Diagnosis should be considered in patients with community acquired pneumonia and exposure to birds. The treatment of choice is a 14 - 21 day course of doxycycline. Complications are uncommon in patients treated with appropriate antibiotics. Amoxicillin-clavulanate (A), levofloxacin (C) and trimethoprim-sulfamethoxazole (D) are not effective in the treatment of psittacosis.

Type 1 diabetes mellitus is the result of which of the following? Destruction of alpha cells in the islets of Langerhans Destruction of beta cells in the islets of Langerhans Destruction of delta cells in the islets of Langerhans The presence of Leydig cells

Correct Answer ( B ) Explanation: The islets of Langerhans in the pancreas contain beta cells that are responsible for producing insulin. Type 1 diabetes mellitus occurs when these cells are destroyed, either through an autoimmune process (type 1A) or a non-autoimmune process (type 1B). Individuals are genetically susceptible to this disorder which is thought to be initiated by environmental triggers. The genetic markers for diabetes mellitus type 1 are present from birth. Type 1 diabetes mellitus is different from type 2 diabetes mellitus. In type 2, decreased insulin release and insulin resistance are responsible for the disorder. Treatment for type 1 diabetes mellitus involves following a proper diet and exercise plan coordinated with insulin replacement therapy. The other cells in the pancreas are not connected with type 1 diabetes mellitus. Alpha cells in the islets of Langerhans (A) produce glucagon. Delta cells (C) produce somatostatin. Leydig cells (D) are found in the testes and are responsible for producing androgens.

Which of the following treatments is most appropriate for a patient with acutely symptomatic sarcoidosis? Lung transplantation Prednisone Theophylline Tiotropium

Correct Answer ( B ) Explanation: The treatment of pulmonary sarcoidosis is based on the underlying pathology. Essentially, sarcoidosis is the result of an excessive immune system reaction. As such, anti-inflammatory medications are logical modalities. In up to 75% of all patients with pulmonary sarcoidosis, non-steroidal anti-inflammatory medications are all that is required to obtain symptom relief. However, 15% of patient with pulmonary sarcoidosis exhibit more severe symptoms and pulmonary decline. The cornerstone of therapy for these patients is oral or inhaled corticosteroids, usually given in a tapered fashion over a 6-month period. Acute flares can usually be treated successfully with 3 week tapering doses. Interestingly, there is not much concrete data to support the benefit of corticosteroid treatment even though it is widely used. Lung transplantation (A) is reserved for end-stage (stage IV) pulmonary sarcoidosis, especially when the FVC drops below 50% or the FEV1 falls below 40% of predicted. Theophylline (C) is recommended for pulmonary hypertension and asthma. Its bronchodilatory action is not effective for sarcoidosis. Tiotropium (D) is a first-line anticholinergic used in the management of emphysema, not sarcoid.

What is the most common historical exposure found in cases of infantile botulism? Construction site dust with botulinum spores Contaminated honey Contaminated infant formula Contaminated wounds

Correct Answer ( B ) Explanation: There are three types of botulism: (1) Infantile where C. botulinum spores are ingested, leading to intestinal tract colonization and toxin release; (2) Foodborne where preformed toxins, usually in canned foods, are ingested; and (3) Wound where a wound is directly exposed to C. botulinum. Infant botulism is the most common type of botulism (> 70 percent of cases). Foodborne and wound botulism are not typically diseases of infancy or childhood. More than 20 percent of infant-type botulism is linked to honey exposure. While infantile botulism is linked to honey exposure, it is not caused by direct spore ingestion and, therefore, is not considered foodborne. More than half of USA cases occur in California, and there is equal distribution between males and females. Clinically, botulism exerts its effects by the toxin it secretes. The toxin acts on pre-synaptic nerve terminals to block the release of acetylcholine by impairing the exocytosis process. This manifests in infants as bulbar palsies, hypotonia, and respiratory insufficiency requiring supportive ventilation. Diagnosis of infantile botulism cannot be made through serologic testing and requires stool culture and toxin assay. It is important to note that antibiotics are contraindicated since they can cause intestinal release of additional toxin. Exposure to construction site dust (A) is a known risk factor for infantile botulism. It is a less frequent cause of botulism than honey exposure. Formula use (C) is not a known risk factor. In fact, breast-fed infants have a higher incidence of botulism than formula-fed infants. Positive wound contamination (D) is the cause of wound botulism. Compared to infantile botulism, wound botulism is much more rare.

A 30-year-old woman presents with fever and abdominal pain. She is 3 days postpartum after cesarean section. Physical examination reveals lower abdominal tenderness to palpation and foul smelling vaginal discharge. What management is indicated? Ceftriaxone IM and Azithromycin PO Clindamycin IV + gentamicin IV Fluconazole Metronidazole

Correct Answer ( B ) Explanation: This patient presents with endometritis and should be treated with broad-spectrum antibiotics and admitted to the hospital. Endometritis affects 1 in 20 vaginal deliveries and 1 in 10 cesarean sections. There are a number of associated risk factors including operative delivery, prolonged rupture of membranes, lack of prenatal care and frequent vaginal examinations. Endometritis is a polymicrobial infection with gram-positive cocci and gram-negative coliforms involved. Patients typically present with abdominal pain, fever and foul-smelling lochia or discharge. It commonly develops the second or third day post partum. Diagnosis is made clinically but ultrasound is recommended to identify any possible retained products of conception. Patients should be treated empirically with broad spectrum antibiotics. Clindamycin IV and gentamicin IV are typically recommended. Although most patients with endometritis are admitted for IV antibiotics (especially those who are ill appearing, have had cesarean section or have underlying illnesses), patients with mild illness may be treated as an outpatient with oral antibiotics and close OB follow up. Ceftriaxone IM and azithromycin PO (A) is used for the treatment of cervicitis. Fluconazole (C) is effective against vaginal yeast infections. Metronidazole (D) alone is effective in the treatment of trichomonas vaginitis.

A 57-year-old man with a history of chronic kidney disease (baseline Cr = 3.3) and liver cirrhosis presents with confusion. Examination reveals scleral icterus and asterixis. Vital signs and serum glucose are normal. What management is indicated? Head CT and lumbar puncture Lactulose Neomycin Protein restriction diet

Correct Answer ( B ) Explanation: This patient presents with hepatic encephalopathy; a state of disordered cerebral function resulting from acute or chronic liver disease. Encephalopathy in these patients is complex and is related to the liver's inability to perform its normal metabolic functions. Ammonia is a marker of this process. Ammonia is formed primarily in the gastrointestinal tract by the action of bacteria on proteins. Ammonia subsequently crosses the blood-brain barrier and leads to increased glutamine levels in the CNS. There are four stages of hepatic encephalopathy. Diagnosis can be aided with laboratory tests. Ammonia is generally elevated but may not correlate with severity of disease. Liver synthetic function (Prothrombin time, albumin) is usually abnormal. Treatment begins with aggressive supportive care including airway, breathing and circulation assessment. Hepatic encephalopathy patients are usually hemodynamically stable but have a high risk of developing gastrointestinal bleeding, electrolyte abnormalities (hypokalemia and hyponatremia) and hypoglycemia all of which can contribute to changes in mental status. Lactulose, an osmotic cathartic agent, is a primary therapy of this disorder. Lactulose is a poorly absorbed sugar, which is metabolized to lactic acid by colonic bacteria. The resultant acidification of stool traps ammonia as ammonium leading to decreased serum levels. Because it is an osmotic agent, lactulose can cause electrolyte abnormalities and fluid shifts. Neomycin (C) is a poorly absorbed aminoglycoside antibiotic. Oral administration leads to reduction of colonic bacteria responsible for ammonia production. However, neomycin should not be used in those with renal dysfunction as it may further impair renal function. A protein restricted diet (D) is a lifestyle modification that can lead to lower ammonia levels but is a long-term management option that will not help in the acute presentation. In patients with altered mental status, a head CT and lumbar puncture (A) are useful in ruling out infectious causes and space-occupying lesions (tumor, blood). However, the patient's clinical presentation of confusion in the setting of liver cirrhosis and asterixis makes these diagnoses highly unlikely.

A 48-year-old man presents with bilateral swollen lower extremities. Which of the following may lead to a false-negative result for proteinuria on a urine dipstick? Alkaline urine Dilute urine Hematuria Prolonged dipstick immersion in urine

Correct Answer ( B ) Explanation: Urine dipstick tests are often performed to evaluate for the presence of proteinuria as a surrogate for impaired renal function. This occurs through a color change of tetrabromophenol blue. There is an approximate relationship between the protein concentration and color intensity; however, reliably positive results occur only at concentrations above 30 mg/dL. As such, dilute urine can generate false-negative results for proteinuria. In contrast, alkaline urine (A), hematuria (C), and prolonged dipstick immersion in urine (D) can all generate false-positive results.

What is the most common pathogen found in infected cat bites? Bartonella henselae Pasteurella multocida Staphylococcus aureus Streptococcus pyogenes

Correct Answer ( B ) Explanation: While only approximately 5% of dog bites become infected, a significant number of cat bites will become infected. Some studies have found up to 70% of cat bites become infected although this may be an overestimation given that many patients only seek treatment after an infection has developed. Cats have narrower, smaller teeth, resulting in most bites being deep puncture wounds. Cat bites are also more likely to be found on the hands which increases the risk of infection as well. Pathogens responsible for infection include those found in the oral flora of the cat as well as skin flora of the patient. The most common pathogen found in infected cat bites is Pasteurella multocida, a virulent facultatively anaerobic gram negative rod. Infection from Pasteurella typically occurs within 24 hours (earlier than cellulitis due to other pathogens) with erythema, warmth, swelling, and pain on examination. Purulent drainage is often seen as well. Prophylaxis with a 3-5 day course of amoxicillin-clavulanate should be prescribed to all patients who present with a cat bite. For those who present with wounds already infected, treatment options include ampicillin-sulbactam, a third-generation cephalosporin, or a fluoroquinolone. Bartonella henselae (A) is the pathogen responsible for cat scratch disease. Staphylococcus aureus (C) and Streptococcus pyogenes (D) have been isolated in infected cat bites, but to a much lesser degree than Pasteurella species.

Which of the following is the most common cause of bladder calculi? Chronic indwelling catheter Hyperparathyroidism Infection of residual bladder urine with urea-splitting organisms Prostatitis

Correct Answer ( C ) Explanation: Bladder stones are much less common than renal or ureteral calculi. In the United States, bladder stones occur almost exclusively in elderly men, often as a complication of other urologic disease. They are usually associated with urinary stasis but can form in healthy individuals without evidence of anatomic defects, strictures, infections, or foreign bodies. The most common cause is infection of residual bladder urine with urea-splitting organisms. Disorders predisposing to the formation of bladder stones include bladder neck obstruction, neurogenic bladder, vesical diverticula, damage from irradiation, and schistosomiasis. Most vesical calculi are formed de novo within the bladder. Renal stones small enough to pass through the ureters are also small enough to pass through a normally functioning bladder and an unobstructed urethra. Clinically, patients with bladder stones most often complain of pain on voiding and hematuria. A classic complaint is the sudden interruption of the urinary stream. The physical exam is usually unremarkable. CT scan is the most sensitive diagnostic study to identify bladder stones. Because a bladder stone is in itself a sign of an underlying problem, removal of the stone and treatment of the underlying abnormality are nearly always indicated. A chronic indwelling catheter (A) is also a common cause of bladder stones, but not the most common. Hyperparathyroidism (B) leads to increased serum levels of calcium and is a predisposing factor to renal stones. However, it is not the most common cause of bladder stones. Prostatitis (D) does not increase the risk of bladder stones. However, prostatic hypertrophy can lead to urinary obstruction and cystitis, which predispose to bladder calculi.

Koilonychia is most consistent with a deficiency in which of the following? Cobalamin Folate Iron Zinc

Correct Answer ( C ) Explanation: Koilonychia are spoon-shaped concave nails that most often occur in children and usually resolve with aging. To determine whether a nail is spooned, perform the "water drop test." Place a drop of water on the nail. If the drop does not slide off, then the nail is spooned. Koilonychia is most commonly associated with Iron deficiency, but may also be seen with diabetes mellitus, Raynaud's or protein deficiency. Cobalamin (A), or vitamin B12, deficiency will cause hyperpigmentation in the nails but not spooning. Folate (B) deficiency can cause onychorrhexis, longitudinal ridges in the nail, or a central nail ridge but not koilonychias. Zinc (D) deficiency is more commonly associated Muehrcke's lines: paired, narrow, white, transverse bands that reflect abnormal nail bed vasculature. When pressure is applied to the nail bed, these narrow lines will disappear. These lines are also commonly seen with liver cirrhosis.

A previously healthy 21-year-old woman presents to your office with a complaint of lower abdominal discomfort and vaginal discharge for the past 2 weeks. She is currently single, but admits to occasionally "fooling around" with men she meets at parties. She expresses concern that she might be pregnant because of increasing tenderness in her lower abdomen for the past 3 days. Which of the following is the most likely diagnosis? Appendicitis Ectopic pregnancy Pelvic inflammatory disease Pregnancy

Correct Answer ( C ) Explanation: Pelvic inflammatory disease is due to an infection that starts in the cervix or vagina and ascends to the endometrium or fallopian tubes or both causing endometritis and salpingitis. The most common organisms that cause PID are Neisseria gonorrhoeae and Chlamydia trachomatis, which are sexually transmitted. Patients with PID often have a yellow, malodorous vaginal discharge, abnormal vaginal bleeding and midline abdominal pain. CDC guidelines advise that empirical treatment be provided to sexually active young women if they have lower abdominal or pelvic pain, if the cause of pain is unidentified, and if the patient has adnexal tenderness, cervical motion tenderness or uterine tenderness. Sequelae of PID include infertility, chronic pelvic pain, ectopic pregnancy and recurrent salpingitis. Appendicitis (A) and ectopic pregnancy (B) are surgical emergencies that must be ruled out in a young female patient with lower abdominal pain, but the patient history of unprotected sexual encounters with multiple partners points to a diagnosis of PID. A thorough sexual and menstrual history should be completed, and a pregnancy test (D) is indicated, however lower abdominal pain and vaginal discharge are not indicative of pregnancy.

An 18-year-old girl presents to the ED with left ankle pain. Earlier in the day she was playing softball and slid into second base and "twisted her ankle." On exam, you note moderate swelling, tenderness, and pain with passive range of motion of the ankle. You do note some abnormal motion when stressing the joint. Which of the following is the most likely diagnosis? First-degree sprain First-degree strain Second-degree sprain Second-degree strain

Correct Answer ( C ) Explanation: Sprains are classified as ligamentous injuries resulting from an abnormal motion of a joint. In such cases, there is injury to the ligamentous fibers of a supporting joint. Sprains are graded according to the severity of pathologic findings; however, clinically the grades are often indistinct. A second-degree sprain is a partial tear of a ligament (more than first-degree). Clinically, there will be moderate hemorrhage and swelling, tenderness, painful motion, abnormal motion, and loss of function. Although there may be some laxity with stressing of the joint, an absence of end points will be seen only with complete ligament rupture (i.e., third-degree sprains). A first-degree sprain (A) is characterized by minor tearing of ligamentous fibers with mild hemorrhage and swelling. There is minimal point tenderness. Stressing the ligament produces some pain, but there is no opening or abnormal joint motion. A strain is an injury to a musculotendinous unit resulting from violent contraction or excessive forcible stretch. Sometimes the term "pulled muscle" is used interchangeably with muscle strain. A first-degree strain (B) is a minor tearing of the musculotendinous unit, characterized by swelling, local tenderness, and minor loss of function. With a second-degree strain (D), more fibers are torn but without complete disruption. There is also greater swelling, ecchymosis, and loss of strength.

A five-year-old child is brought to clinic with scalp itching. Adherent white flecks are noted around hair proximal to the scalp. What is the treatment of choice? Ketoconazole shampoo Oral ivermectin Topical 1% permethrin Topical lindane

Correct Answer ( C ) Explanation: The child has classic findings of pediculosis capitis, also known as head lice, including nits and intense itching. Pediculosis is caused by infestation with Pediculus humanus capitis, the human head louse. The lice are spread by direct contact or through fomites and can be difficult to eradicate. The mature female louse lays eggs, known as nits, which are strongly adherent to the hair shaft. The treatment of choice for pediculosis capitis is a topical insecticide. An example is topical 1% permethrin, which is applied to saturate wet, freshly shampooed hair for ten minutes prior to rinsing. Treatment should be repeated in seven to 10 days. Although 5% permethrin is also available by prescription, it has not been shown to be more efficacious than 1% permethrin. Other topical treatment options include malathion, benzyl alcohol, spinosad, and ivermectin. Topical lindane (D) is an alternative topical insecticide. However, due to rare reports of neurotoxicity, it is not recommended as first-line therapy. Oral ivermectin (B) is also an effective treatment for pediculosis, but it is recommended only for children who have failed topical therapy. Ketoconazole shampoo (A) is used for the treatment seborrheic dermatitis of the scalp, but it is not an effective treatment for pediculosis. Notably, seborrheic dermatitis may result in hair casts, which are whitish, desquamated skin. Although hair casts may initially appear like nits, they can be distinguished by their ability to slide easily along hair shafts.

Which of the following tests for function of the median nerve? Abduction of the index finger against resistance Extension of the wrist against resistance Pincer function of the thumb and index finger Sensation of the dorsum of the first webspace

Correct Answer ( C ) Explanation: The median nerve, which is derived from the medial and lateral cords of the brachial plexus, contains nerve roots from C5-T1. In the forearm, it gives rise to the anterior interosseous nerve, which supplies the deep muscles of the anterior forearm, and the palmar cutaneous nerve which innervates the skin of the lateral aspect of the palm. It then crosses through the carpal tunnel terminating as the recurrent branch, responsible for innervating the thenar muscles, and the palmar digital branch. The palmar digital branch innervates the palmar surface and fingertips of the thumb, index, and middle digits, as well as the lateral two lumbrical muscles. Motor function of the median nerve can be tested by having the patient make an "OK" sign or testing the pincer function of the thumb and index finger. Sensation at the volar tip of the index finger confirms sensory function of the median nerve. The ulnar nerve is responsible for abduction of the index finger against resistance (A). Extension of the wrist against resistance (B) and sensation of the dorsum of the first webspace (D) is a function of the radial nerve.

A 9-year-old boy presents to the ED with bilateral knee pain, low-grade fever, nausea, vomiting, and diarrhea for the past 4 days. His vital signs are blood pressure of 116/80 mm Hg, heart rate of 98 beats per minute, respiratory rate of 14 breaths per minute, and a temperature of 38.1°C. On examination, you note the rash seen above. Urinalysis is positive for hematuria. Which of the following statements is the most accurate? Despite plasma exchange, most patients progress to chronic renal impairment Long-term prednisone therapy improves 5-year survival to greater than 50% The disease is self-limited; most cases resolve within 6-8 weeks Without treatment, the disease carries a mortality rate of 80% at one year

Correct Answer ( C ) Explanation: The patient has Henoch-Schönlein purpura (HSP). This small-vessel vasculitis predominantly occurs in small children. Most cases follow an upper respiratory tract infection. HSP classically presents with fever, abdominal pain, arthritis, hematuria, and a pathognomonic round, palpable, symmetrical rash that appears on the dependent areas of the legs and buttocks. NSAIDs, dapsone, and prednisone have all been shown to relieve symptoms. The course of disease is typically self-limited. Most cases resolve within 6 to 8 weeks, with a recurrence rate of up to 33%. Plasma exchange (A) has been found to be successful in the treatment of microscopic polyangiitis. In HSP, most patients do not progress to chronic renal impairment. Prednisone (B) therapy has increased the 5-year survival rate to greater than 50% in Churg-Strauss Syndrome, which is typically associated with fever, weight loss, malaise, and pulmonary symptoms, but it does not affect survival in HSP. Granulomatosis with polyangiitis (D), not HSP, once carried a 1-year mortality rate of 80%, however, the combination of cyclophosphamide and corticosteroids has been successful in inducing remission in more than 90% of patients.

Which of the following is an absolute contraindication to the diphtheria vaccine? Family history of sudden infant death syndrome Moderate or severe illness Previous anaphylaxis to DTaP Previous seizure within three days of last DTaP dose

Correct Answer ( C ) Explanation: The routine childhood immunization schedule in the United States includes vaccination against diphtheria, tetanus and pertussis. There has been a dramatic decrease in the incidence of diphtheria since the initiation of the vaccine in the 1940's. Corynebacterium diphtheriae causes acute cutaneous or respiratory illness and can be fatal. Mortality is higher in children under the age of five. The recommended schedule is five doses of the diphtheria, tetanus and pertussis (DTaP) vaccine at ages 2 months, 4 months, 6 months, 15-18 months and 4-6 years. The tetanus toxoid, reduced diphtheria toxoid, and acellular pertussis vaccine (TDaP) is used as a single booster dose for adolescents or adults. Absolute contraindications to the DTaP vaccine include previous anaphylaxis to DTaP, anaphylactic reactions to latex, and progressive neurologic disorder and encephalopathy within seven days of a previous dose of DTaP without identifiable cause. A family history of sudden infant death syndrome (A) is not an absolute contraindication to administration of DTaP. A number of factors merit precautions in administration of the DTaP vaccine, but are not contraindications. These include moderate or severe illness (B), previous seizure within three days of last dose (D), a temperature of greater than 104°F within 48 hours of previous dose, and a history of Guillain-Barré syndrome.

A 7-year-old boy presents for a well child check. His mother reports that he has always been a healthy child and does well in school. On examination, he is noted to have pectus excavatum and long, tapered fingers. He is wearing glasses, and mother reports a history of upward lens subluxation. What is the most likely genetic abnormality? Ehlers Danlos Syndrome Homocystinuria Marfan Syndrome Osteogenesis Imperfecta

Correct Answer ( C ) Explanation: This child has findings concerning for undiagnosed Marfan Syndrome. Marfan Syndrome is an autosomal dominant connective tissue disorder with a broad range of clinical severity. The condition is caused by a mutation in fibrillin-1 (FBN1) gene. Features of Marfan syndrome include aortic root dilation or dissection, mitral valve prolapse, excessive linear growth of long bones, joint laxity, arachnodactyly, pectus deformities, scoliosis, kyphosis, hindfoot valgus, decreased upper segment to lower segment ratio, increased arm span to height ratio, ectopia lentis, emphysematous pulmonary changes, spontaneous pneumothorax, skin striae, and dural ectasia. The revised Ghent Criteria are used to establish a diagnosis and include combinations of aortic disease, ectopia lentis, FBN1 mutations, family history, and a systemic score. The systemic score includes items such as pectus carinatum, hindfoot deformities, pneumothorax, scoliosis, and facial features. Lens subluxation is typically upward in Marfan Syndrome. Ehlers Danlos Syndrome (A) is a rare connective tissue disorder. Primary clinical features include skin hyperextensibility, tissue fragility, and hypermobility. In the case of the above patient, pectus excavatum and arachnodactyly should raise concern for Marfan Syndrome rather than Ehlers Danlos Syndrome. Homocystinuria (B) is an autosomal recessive disorder of homocysteine metabolism that results in a Marfanoid body habitus, along with ectopia lentis. The lens subluxation in homocystinuria is typically downward, in contrast to the typical upward subluxation in Marfan Syndrome. Additionally, patients with homocystinuria typically have significant intellectual disability, thromboembolic disease, and severe atherosclerosis. Osteogenesis imperfecta (D) is a connective tissue disorder that presents with a wide spectrum of clinical phenotypes, ranging from severely brittle bones in infancy to premature osteoporosis later in life. The diagnosis should be suspected in children with multiple or atypical fractures. Other clinical features may include short stature, scoliosis, blue sclerae, hearing loss, hyperextensibility, and easy bruising. Arachnodactyly and lens subluxation are not common features of Osteogenesis Imperfecta.

A 43-year-old woman with breast cancer on chemotherapy presents with a fever to 102°F. She also complains of a cough and generalized fatigue. Physical examination and chest X-ray are unremarkable except for the presence of a mediport. Complete blood count reveals a white blood count of 600 with 30% neutrophils and no band forms. What management is indicated? Administer filgrastim and discharge home Draw blood cultures and await results for treatment Send blood and urine cultures and start vancomycin and cefepime Start levofloxacin and admit for pneumonia

Correct Answer ( C ) Explanation: This patient presents with neutropenic fever and requires culture of typical infectious sources and broad-spectrum antibiotics. Fever can be caused by a number of etiologies in cancer patients including inflammation, medications, antimicrobials, transfusions and tumor necrosis. Most fevers occurring in cancer patients are infectious in origin (55-70%). Neutropenia is defined as an absolute neutrophil count (ANC) < 500 cells/mm3 (or <1000 cells mm3 with predicted decline to < 500 cells/mm3) and is calculated by multiplying the total WBC count by the percentage of neutrophils and bands. Neutropenic fever is an oncologic emergency and is defined by neutropenia along with a single oral temperature >101°F or a temperature of 100.4°F for at least 1 hour. All of these patients should be managed presuming that they have a serious bacterial infection regardless of the presentation. A thorough physical examination should be performed looking for an infectious source but spontaneous bacteremia is common. Common sites include urine, chest, skin and blood and cultures should be obtained. Patients with chronic indwelling catheters should have a culture sent from this site. Broad-spectrum antibiotics covering the most likely pathogens should be started. An antipseudomonal penicillin with an aminoglycoside or a fourth generation cephalosporin are commonly started. Vancomycin should be added in patients with possible methicillin resistant Staphylococcus aureus (MRSA). This includes patients with recent hospitalizations and indwelling catheters or severe illness. Untreated neutropenic fever has a high mortality rate (~ 20%). Filgrastim (A) is used by oncologists to boost the white blood cell count in patients undergoing chemotherapy but has a delayed onset of action and will not treat a serious bacterial infection. All patients with neutropenic fever should have antibiotics started and waiting for culture results (B) delays care. Although the patient has symptoms concerning for pneumonia, levofloxacin (D) is not broad enough to cover the likely pathogens that may be present in neutropenic fever.

A 35-year-old woman comes to the clinic complaining of difficulty seeing, blurred vision, eye pain, and cough. She describes the cough as being dry and nonproductive. She has no past medical history and takes no medications. Ophthalmologic examination shows uveitis. Chest X-ray reveals bilateral hilar adenopathy. Which of the following laboratory findings would also most likely be associated with this patient's condition? Elevated C-reactive protein Elevated ESR Elevated serum angiotensin converting enzyme levels Increased sweat chloride

Correct Answer ( C ) Explanation: Uveitis, bilateral hilar adenopathy, and a dry cough most likely indicate sarcoidosis. Sarcoidosis is granulomatous disease that can form nodules in multiple organs. Patients with sarcoidosis typically present with fatigue, weight loss, arthritis, dry eyes, blurry vision, and respiratory symptoms (eg, cough, dyspnea). Management usually involves the use of corticosteroids (eg, prednisone). Elevated levels of serum angiotensin converting enzyme (ACE) are typically elevated in about 75% of patients with sarcoidosis. However, elevated levels of ACE can also be seen in other conditions, such as, asbestosis, diabetes, and lung cancer. Therefore, it is not routinely used as a diagnostic test. Although, levels of C-reactive protein (CRP) (A) and erythrocyte sedimentation rate (ESR) (B) are mildly elevated in patients with sarcoidosis, they are not useful in assessing disease activity nor differentiating sarcoidosis from other causes of inflammation. Increased sweat chloride (D) is typically seen in patients with cystic fibrosis.

A 42-year-old woman presents for evaluation of irregular menses, associated with episodes of excessive daytime sweating and palpitations. She has not had a hysterectomy or oophorectomy. She is currently sexually active, although intercourse has been more uncomfortable lately. Which of the following disorders would you consider in this patient's differential diagnosis? Hyperaldosteronism Hyperprolactinemia Hyperthyroidism Hypokalemia

Correct Answer ( C ) Explanation: When considering irregular menses in a middle-aged woman, a clinician should remember there are conditions other than menopause which can be causative. In any woman with changes in menstruation, consider pregnancy, and obtain a human chorionic gonatropin (hCG) level. A woman over age 45 who presents with menstrual irregularities plus common menopausal symptoms (such as palpitations, hot flashes, vaginal dryness and dyspareunia, as in the patient above), is more likely transitioning into menopause than presenting with a new endocrine disorder, and as such, no further diagnostic evaluation is recommended. For women under 45 years of age with menstrual irregularities, consider hyperthyroidism, especially if the patient also demonstrates mood changes and hot flash type symptoms, like sweating. Hyperprolactinemia (B) also causes menstrual irregularities, especially if galactorrhea is also present. Consider premature ovarian failure for women under 40 years of age with irregular menses. Hyperaldosteronism (A) is an adrenal gland disorder that typically causes hypertension and hypokalemia. Aldosterone does not affect the menstrual cycle. Hypokalemia (D) must be considered in any patient with palpitations, as it can lead to premature atrial and ventricular beats, atrioventricular block and paroxysmal atrial tachycardia, among other cardiac arrhythmias. However, menstrual irregularities are not part of hypokalemic symptomatology.

A 22-year-old woman presents with pain and swelling to the vulva. On examination, you notice an area of swelling with induration and central fluctuance at the 8 o'clock position. Which of the following statements is true regarding this? All patients require antibiotics Gonorrhea and chlamydia are the most common causes Incision should be performed in the operating room Word catheter is placed for four to six weeks

Correct Answer ( D ) Explanation: A Bartholin's abscess occurs with infection of an obstructed Bartholin gland, a pea-sized mucous secreting gland located on each side of the labia minora in the 4 and 8 o'clock positions. When the duct that drains fluid from the gland becomes blocked, a mucous-filled cyst forms and eventually, if not drained, bacterial overgrowth occurs leading to an abscess. It is also possible to have a primary infection of the Bartholin gland itself. Most commonly, the causative organism is normal vaginal flora of which E. coli is most prevalent in cultures. Treatment of the abscess is with incision and drainage. After the loculations are broken up and all contents of the abscess have been expressed, either gauze packing or a Word catheter is placed. Gauze packing is removed after 24 to 48 hours and patients may require marsupialization at a later date. The Word catheter is a plastic catheter with a balloon on the end that is filled with 2 to 4mL of water and expanded within the abscess cavity. It is important that the incision is made only slightly larger than the opening required for catheter placement. The catheter remains in place for four to six weeks and patients are instructed to abstain from vaginal intercourse. This duration is required for epithelialization of the tract and the formation of a small fistula tract that will allow appropriate drainage without obstruction. All patients do not require antibiotics (A). Like other abscesses, if adequate drainage is obtained in the immunocompetent host, antibiotics are not necessary. Many gynecologists recommend antibiotic therapy if cellulitis is present. Gonorrhea and chlamydia (B) are not the most common causes. Together they are responsible for approximately 10% of these abscesses. Incision is not performed in the OR (C) unless there is larger infection requiring anesthesia for facilitation of the drainage. Patients with recurrent abscesses may need marsupialization performed in the OR but this is not during the acute infection.

Which of the following is a complication of an untreated mallet finger injury? Boutonniere deformity Dupuytren's contracture Jersey finger Swan-neck deformity

Correct Answer ( D ) Explanation: A mallet finger is caused by a forced flexion of the distal interphalangeal joint (DIP) leading to rupture of the extensor tendon at its insertion at the base of the distal phalanx or bony avulsion of the tendon insertion site. Appropriate treatment includes splinting the distal interphalangeal joint in full extension or slight hyperextension while allowing the proximal interphalangeal joint to have full range of motion. In untreated or under treated cases, a swan neck deformity occurs. Other causes include rheumatoid arthritis and connective tissue disorders (Ehlers-Danlos syndrome). A swan neck deformity is characterized by hyperextension of the proximal interphalangeal joint (PIP) and flexion of the distal interphalangeal joint (DIP). Dupuytren's contracture (B) refers to a thickened and fibrous palmar fascia which is due to fibrous proliferation. It occurs commonly in men over age 40 years, but it also is common in those of Northern European descent, diabetics, alcoholics and epileptics. Boutonniere deformity (A) occurs from forced flexion at the proximal interphalangeal joint (PIP), causing a tear of the central portion of the extensor tendon at the proximal interphalangeal joint (PIP). Patients are unable to fully extend at the proximal interphalangeal joint (PIP) with the wrist and metacarpalphalangeal joints fully extended. Jersey finger (C) occurs in cases such as a football player grabs another player's jersey, avulsing the profundus tendon from its bony insertion.

A 22-year-old man presents with 4 days of bilateral facial enlargement and fever. These symptoms began insidiously and have persisted. Examination reveals generalized warmth and tenderness just anterior to the ears near the temporomandibular joints. He has not had any recent travel, but he did live in Africa between the ages of 3 and 7. Which of the following is the most likely diagnosis? Bacterial dacrocystitis Bacterial parotitis Viral dacrocystitis Viral parotitis

Correct Answer ( D ) Explanation: Acute viral parotid gland infection is called parotitis, or mumps. Transmission of the paramyxovirus is by direct spread of oropharyngeal secretions. Mumps is classified by nonprogressive parotid gland pain and swelling which usually resolves within nine days of onset. Associated symptoms include fever, malaise and anorexia. Bilateral involvement is usual. Mumps is rare in developed countries because of immunization at one year and 4-6 years of age. Outbreaks are possible however in teenagers and young adults who never received the second vaccine, which is possible in people who lived abroad between the ages of 4 and 6 years. Symptoms include local pain, erythema and edema, ipsilateral otalgia and uncomfortable chewing. Treatment is supportive. Rarely, mumps will lead to meningoencephalitis, orchitis, pancreatitis and deafness. Dacrocystitis (A and C) is inflammation and infection of the lacrimal glands or tear ducts or both. Bacterial parotitis (B) is caused by ascending bacteria from the mouth. It most frequently occurs in the elderly or chronically ill. A key to distinguishing a viral etiology over a bacterial etiology is that viral parotitis is typified by a nonprogressive and short-lived course in otherwise healthy, and typically younger, adult.

A 30-year-old woman presents to your office requesting medication to help her quit smoking. Which of the following is a contraindication to prescribing bupropion? Cardiovascular disease Chronic obstructive pulmonary disease Depression Seizure disorder

Correct Answer ( D ) Explanation: Bupropion is one of two pharmacologic agents used for smoking cessation, the other being varenicline. Bupropion is an antidepressant that is believed to enhance central nervous system noradrenergic and dopaminergic release. Patients start the medication one week prior to their quit date then continue the medication for eleven more weeks. Side effects of bupropion include headache, insomnia, agitation and dry mouth. Bupropion decreases the seizure threshold and is therefore contraindicated in patients with seizure disorder or any predisposition to seizures. Patients with stable cardiovascular disease (A) and chronic obstructive pulmonary disease (B) may safely use bupropion. Bupropion is an antidepressant used in patients with depression (C). Bupropion has been associated with an increased risk of suicidal behavior and depression, but less so than varenicline. Discussion with patients about the risks and benefits of this medication should occur and patients should be advised to seek medical attention for any unusual mood symptoms or behavior.

You are treating a patient who is admitted to the hospital with a severe flare of ulcerative colitis. You have prescribed antibiotics, corticosteroids, and a biologic medication only. Which of the following is this patient most at risk for in this acute phase? Colon cancer Pancreatitis Peptic ulcer Toxic megacolon

Correct Answer ( D ) Explanation: Inflammatory bowel disease consists of ulcerative colitis (UC) and Crohn's disease (CD). In the US, UC carries an incidence of 1 in 1000 patients while CD occurs in 1 in 3000 people. Both UC and CD frequently present in the 15-30 year old age range, but only CD occurs at a second common time period, also peaking in those 50-70 years of age. Both diseases can be tied to genetic predispositions. Ulcerative colitis is characterized by an inflammation of the colonic mucosa, not transmural inflammation of the GI tract which occurs in Crohn's disease. The main symptoms of UC are bloody diarrhea, tenesmus and lower abdominal cramps. A severe colitis occurs up to 25% of the time, and is characterized by rapid progression, fever, hypotension, decreased hematocrit, >6 bloody bowel movements per day and absent bowel sounds. Colonoscopy findings include continuous inflammation from rectum proximally to other colonic areas. The clinician must rule out any infectious cause before treating an acute flare. Treatment options include corticosteroids, biologics such as anti-tumor-necrosis-factor alpha, mesalamine, olsalazine or sulfasalazine. Excision with J-pouch anal anastomosis is necessary in refractory cases. Toxic megacolon or strictures are complications in up to 5% of patients. Toxic megacolon is a state of colonic atony and systemic toxicity in which the colon dilates >6 cm on a KUB radiograph. This carries a high risk of colonic perforation. It is treated with steroids and antibiotics, and if no improvement in 3 days, surgery may be required. The risk of colon cancer (A) is increased in patients with ulcerative colitis ( 2% at 10 years, 8% at 20 years and 18% at 30 years). Pancreatitis (B) and peptic ulcer (C) are not direct complications of ulcerative colitis.

A 22-year-old woman at 36 weeks gestation presents with complaints of feeling a sudden gush of water coming from her vagina. She has had regular obstetrical follow-up and a normally progressing pregnancy. Which of the following is the most appropriate next step in management? Administration of corticosteroids Immediate digital examination of the cervix Initiation of magnesium sulfate infusion Microscopic evaluation of vaginal fluid

Correct Answer ( D ) Explanation: Premature rupture of membranes (PROM) is the rupture of the fetal membranes before the onset of labor. In most cases, this occurs near term. When membrane rupture occurs before 37 weeks gestation, it is known as preterm PROM. Preterm PROM occurs in 3% of pregnancies and is the cause of approximately 33% of preterm deliveries. It can lead to significant perinatal morbidity, including respiratory distress syndrome, neonatal sepsis, umbilical cord prolapse, placental abruption, and fetal death. Appropriate evaluation and management are important for improving neonatal outcomes. Although occasionally confused with urinary incontinence or excess vaginal fluid, a history of "my water broke" is a fairly reliable indicator of PROM. On exam, a moist perineum with a watery fluid pooling in the vaginal vault will be noted. The presence of amniotic fluid can be confirmed with nitrazine testing (a pH of 7.1-7.3 as opposed to a normal pH of 3.5-6.0), the presence of ferning on microscopic evaluation, or smear combustion (amniotic fluid turns white and crystalizes when Corticosteroids (A) can accelerate fetal lung maturation and are widely used in preterm labor. However, their routine use in PROM has not been established because rupture of the membranes also stimulates fetal lung development. Digital manipulation of the cervix (B) should be avoided because the incidence of infection increases with the number of examinations. Magnesium sulfate (C) inhibits calcium uptake by smooth muscle cells and is used as a tocolytic agent in preterm labor. However, it has not yet been established that this patient is in preterm labor and tocolysis should not be initiated at this time. In addition, any initiation of tocolysis should be coordinated with the receiving obstetrician.

Which of the following is an example of primary prevention? Eliminating dust and pollen for an asthmatic Ophthalmoscope exam in diabetics Papanicolaou test Varicella vaccine

Correct Answer ( D ) Explanation: Prevention traditionally has been divided into three categories: primary, secondary, and tertiary. Primary prevention targets individuals who may be at risk to develop a medical condition and intervenes to prevent the onset of that condition. Examples include childhood vaccinations, water fluoridation, smoking cessation, and education about safe sex. The varicella vaccine is an example of primary prevention. Secondary prevention includes procedures that detect and treat pre-clinical pathological changes and thereby control disease progression. Screening procedures, such as mammography to detect early stage breast cancer, are often the first step, leading to early interventions that are more cost effective than intervening once symptoms appear. Screening is usually undertaken by health professionals, such as routine blood pressure checks or via public health screening programs such as osteoporosis screening. Colonoscopy and Papanicolau test (C) are other common examples of secondary prevention. Tertiary prevention targets individuals with a known disease, with the goal of limiting or preventing future complications. It aids in softening the impact caused by the disease on the patient's function, longevity, and quality of life. Examples include screening diabetics for microalbuminuria, rigorous treatment of diabetes mellitus, and post-myocardial infarction prophylaxis with beta-blockers and aspirin. An annual ophthalmoscope examine in diabetic patients (B) to screen for retinopathy is an example of tertiary prevention. Tertiary prevention can include modifying risk factors, such as assisting a cardiac patient to lose weight, or making environmental modifications to reduce an asthmatic patient's exposure to allergens (A).

Which of the following is the recommended treatment for a pituitary adenoma >1 cm in size? Bromocriptine (Parlodel®) Pergolide (Permax®) Radiation Surgical resection

Correct Answer ( D ) Explanation: Surgical resection is the treatment of choice for pituitary adenomas >1 cm in diameter or those with compression of the optic chiasm, erosion of bone, or extension into the walls of the sella. The transsphenoidal approach (Hardy procedure), preferable to the transcranial route (most often right perioral craniotomy), achieves gross total resection in one-third of patients. Improvement in microsurgical technique and imaging has reduced mortality to below 2%. Dopamine agonists such as bromocriptine (A), cabergoline, quinagolide, and pergolide (B) are effective in micro- as well as macroprolactinomas and lead to reduction in tumor size, improvement of symptoms, and normalization of prolactin levels. Radiation therapy (C) is provided as primary treatment to older adult patients, those who are not surgical candidates, and following partial resection. Tumor shrinkage is seen only years after treatment. Adverse effects are infrequent and include necrosis of the adjacent portions of the temporal lobe, hearing loss, optic neuropathy, and radiation-induced sarcomas.

A 15-year-old girl is in your clinic with her mother for a sports physical examination. She denies any symptoms and is doing well in school. There is no family history of sudden death or heart disease. She has become a vegan for the past year and takes food supplements. She has regular menstrual period. Physical examination is normal. You perform a screening complete blood count (CBC) that reveals hemoglobin of 10 g/dL, mean corpuscular volume (MCV) of 68, with ferritin of 11 ng/mL (normal for females 12-150 ng/mL). Which of the following is the next best step in management? Increase intake of meat, grains, fruits, and vegetables Recheck CBC in one month Start parenteral iron Trial treatment with oral iron supplements

Correct Answer ( D ) Explanation: The adolescent in the vignette has iron deficiency anemia (IDA) that her vegan diet most likely causes. Adolescence is a time of increased iron needs because of the expansion of blood volume and increases in muscle mass. The incidence of iron deficiency among adolescents appears to be rising and those at particular risk are adolescents who limit their intake of meat products. The primary laboratory values obtained when evaluating an adolescent for iron deficiency include a complete blood count with red blood cell indices and serum ferritin. In most cases, iron deficiency is identified by a low serum ferritin concentration and iron deficiency anemia by a hemoglobin concentration below 11.0 g/dL combined with a serum ferritin less than 12 ng/mL. The anemia typically is microcytic and hypochromic. The treatment of iron deficiency for patients with mild or moderate degrees of iron deficiency (hemoglobin > 9 g/dL) involves a trial of treatment with oral iron supplements (3 to 6 mg of elemental iron/kg/day). Follow up monitoring should be performed to ensure response to the supplementation, which also helps to confirm the diagnosis of iron deficiency. Increasing intake of meat, grains, fruits, and vegetables (A) is inappropriate for the girl who is vegan and prefers not to eat meat. Rechecking CBC in one month (B) is not correct because the girl has IDA and needs treatment. Starting parenteral iron (C) should be reserved for patients with severe, persistent anemia who have proven intolerance to oral supplements, malabsorption, or poor compliance to oral therapy, which is not the case for the girl in the vignette.

A two-year-old girl is brought to your clinic by her father because of abdominal pain. This is accompanied by decreased energy, vomiting, and constipation. CBC with peripheral smear shows microcytic anemia with basophilic stippling. Which of the following is the most likely diagnosis? Acute lymphocytic leukemia Folate deficiency Iron deficiency anemia Lead poisoning

Correct Answer ( D ) Explanation: The girl's symptoms and laboratory value are most likely due to lead poisoning. Since lead blocks iron from being incorporated into heme, lead poisoning can look like iron deficiency in which there is microcytic anemia. To differentiate this from iron deficiency, look for basophilic stippling of RBCs accompanied by gastrointestinal (GI) and central nervous system (CNS) symptoms. GI symptoms of lead poisoning include anorexia, abdominal pain, vomiting, and constipation, often occurring and recurring over a period of weeks. CNS symptoms are related to worsening cerebral edema and increased intracranial pressure. These symptoms include headaches, change in mentation, lethargy, papilledema, seizures, and coma. Lead lines are found on imaging of long bones. The most common pathway by which lead enters the body is through nonnutritive hand-to-mouth activity of young children. In most cases, lead is ingested, either as a component of dust licked off surfaces or in swallowed paint chips, through water contaminated by its flow through lead pipes or brass fixtures, or from food or liquids contaminated by contact with lead-glazed ceramic ware. In acute lymphocytic leukemia (A), symptoms include fever, lymphadenopathy or joint pain. The CBC would either show a very high leukocyte count with lymphocyte predominance or may present with pancytopenia. Folate deficiency (B) is characterized by macrocytic anemia. Iron deficiency anemia (C) presents with microcytic anemia but there is no basophilic stippling.

A 54-year-old man presents to the ED complaining of back pain. He states that over the previous two days he developed a headache, cough, and runny nose. You perform a physical examination and note lesions as seen in the picture above. What is the cause of this skin condition? Herpes simplex virus Morbillivirus Roseolovirus Varicella-zoster virus Variola

Correct Answer ( D ) Explanation: The rash is consistent with shingles, which is due to reactivation of the latent varicella-zoster virus. This condition is most often seen in older individuals. The classic rash is dermatomal in distribution and is often described as grouped vesicles on an erythematous base. The lesions remain in congruent stages of healing as compared to chicken pox (varicella) that exhibits multiple stages of healing. Infection begins as a prodrome of headache, photophobia, malaise, and itching and burning in the affected area 1-3 days before the appearance of the rash. Herpes simplex virus (A) is responsible for similar skin lesions defined by their small, thin-walled grouped vesicles on an erythematous base. Herpes simplex I typically causes oral lesions, whereas herpes simplex 2 mainly causes genital lesions. The virus that causes measles (B) is part of the Morbillivirus genus. The characteristic measles rash is described as a generalized, maculopapular, erythematous rash that begins several days after the onset of fever. It starts on the head before spreading to cover most of the body, often causing itching. The rash is said to change color from red to dark brown before disappearing. Roseolovirus (C) is responsible for the childhood condition roseola. Roseola is characterized by several days of high fever that resolves just as a pale, pink macular rash develops on the face, neck, and chest. Variola (E) is the causative agent for smallpox. This rash begins as small red spots on the tongue and in the mouth and develops into sores that spontaneously rupture. Around the third day, the rash evolves into diffuse, raised bumps with subsequent appearance of pustules that are round, sharply defined, and firm to the touch.

The parents of a 7-year-old boy ask you to evaluate him because of increasing concerns about his temper tantrums over the past 9 months. He becomes angry and hostile, argues constantly, and refuses to follow rules or directions. The boy often becomes aggressive and destructive, breaking his toys and sweeping his dinner plate and glass of milk onto the floor. The parents believe that their son is deliberately behaving this way to annoy them. This history is most consistent with which one of the following? Antisocial personality disorder Bipolar disorder Conduct disorder Oppositional defiant disorder

Correct Answer ( D ) Explanation: This child meets the DSM-V criteria for oppositional defiant disorder, defined as a pattern of negative, hostile, and defiant behavior lasting at least 6 months. The child will often lose his or her temper, argue with adults, actively defy or refuse to comply with adults' requests or rules, deliberately annoy people, blame others for their mistakes or misbehavior, be easily annoyed by others, appear angry and resentful, or be spiteful or vindictive. The disturbance in behavior must also cause clinically significant impairment in social, academic, or occupational functioning, and the behaviors must not occur exclusively during the course of a psychotic or mood disorder. Coexisting conditions are common in children with oppositional defiant disorder, particularly ADHD and mood disorders. Research supports outpatient psychological interventions for children with oppositional defiant disorder. Studies have demonstrated that parent training is an effective means of reducing disruptive behavior. Conduct disorder (C) involves more deliberate aggression, destruction, deceit, and serious rule violations, such as staying out all night, animal cruelty, bullying, burglary or chronic school truancy. Meeting the criteria for conduct disorder excludes the diagnosis of oppositional defiant disorder. When conduct disorder is carried into adulthood, it is known as antisocial personality disorder (A). Individuals with antisocial personality disorder display a pervasive pattern of disregard for and violation of the rights of others and the rules of society. The diagnosis of antisocial personality disorder is made only after one is aged at least 18 years, however the features must start to be exhibited by age 15 years or earlier. Bipolar disorder (B) is characterized by symptoms that include periods of mania, hypomania, psychosis, or depression interspersed with periods of relative wellness. Defiance and hostile behavior is not generally seen in bipolar disorder

A 40-year-old woman, who actively uses intravenous drugs, presents to the ED with fever and fatigue for the past 3 days. In the ED, her vital signs are BP 126/82, HR 90, RR 16, oxygen saturation 99% on room air, and temperature 101.6°F. On exam, a murmur is noted. A transesophageal echocardiography is ordered for suspected endocarditis. Which of the following valves is most likely to be affected? Aortic Mitral Pulmonic Tricuspid

Correct Answer ( D ) Explanation: This patient most likely has bacterial endocarditis. Endocarditis is more common in patients with valvular abnormalities, prosthetic valves, and IV drug users. Common pathogens include Staphylococcus aureus and viridans group streptococci. Of note, viridans streptococcus endocarditis commonly presents after dental work. Endocarditis presents most commonly with fever and malaise, although other signs and symptoms may be present. Although the classic triad is fever, anemia, and a heart murmur, this rarely presents clinically. IV drug users usually have right-sided endocarditis, most commonly affecting the tricuspid valve. The murmur noted on exam is usually tricuspid regurgitation. Labs should be ordered for those with suspected endocarditis, as one would expect leukocytosis, anemia, increased ESR, and increased CRP. Three blood cultures from three separate venipuncture sites should be taken, with the first and last draw occurring at least one hour apart. The most useful diagnostic imaging is an echocardiogram, especially a TEE, demonstrating vegetations. The Duke criteria are the most widely used and accepted criteria to clinically diagnose bacterial endocarditis. Empiric antibiotics should be initiated in those with suspected or confirmed endocarditis once cultures have been drawn, with antibiotics being altered as needed once the pathogen and susceptibility return. Left side valves (aortic, mitral) are most commonly involved in endocarditis outside of intravenous drug abusers and prosthetic valves. The mitral valve (B) is the most commonly affected valve in infective endocarditis, although the aortic valve (A) may also be affected. The pulmonic valve (C) is much less commonly affected.

A 32-year-old woman presents with right eye pain and a foreign body sensation. A fluorescein enhancement is shown above. Which of the following is most likely indicated? Acyclovir and follow up Eye patching and follow up Lid eversion, oral antibiotics and follow up Lid eversion, topical antibiotics and follow up

Correct Answer ( D ) Explanation: This patient presents with a traumatic corneal abrasion requiring topical antibiotics and follow up. It is also important to perform lid eversion on all patients with corneal abrasion to ensure that the foreign body that caused the abrasion is no longer present. Corneal abrasions are common and cause pain, photophobia, deceased visual acuity and a foreign body sensation. A hallmark of corneal abrasion is the complete or near-complete relief of pain with topical anesthetic drops. Physical examination often reveals conjunctival injection and may show a gross abrasion. Addition of fluorescein and viewing with a cobalt blue light can enhance the abrasion. If the abrasion lies over the visual axis, acuity can be significantly reduced. Treatment involves careful inspection for any retained foreign body including under the eyelid. Antibiotic ointment or drops should be employed and follow up should be scheduled with an ophthalmologist in 24-48 hours. Patients who wear contacts require special consideration. Antibiotic drops in these patients should have pseudomonal coverage and contacts should not be worn until cleared by an ophthalmologist. Acyclovir (A) is required for the treatment of herpes keratitis, which forms a branching pattern defect on the cornea. Eye patching (B) should be avoided. Oral antibiotics (C) are not necessary for simple corneal abrasions.

A 3-day-old baby girl has a holosystolic murmur along the left lower sternal border. What is the most likely diagnosis? Aortic stenosis Atrial septal defect Patent ductus arteriosus Ventricular septal defect

Correct Answer ( D ) Explanation: Ventricular septal defects (VSD) present with a holosystolic murmur best heard at the left lower sternal border and is frequently accompanied by a thrill or displaced point of maximal impulse. A defect in the membranous (superior) portion of the ventricular septum is more common than in the muscular (inferior) portion. Auscultation is often sufficient to make the diagnosis of a VSD but confirmation can be obtained with echocardiography. The clinical findings of patients with a VSD vary according to the size of the defect and pulmonary blood flow and pressure. Small VSDs with trivial left to right shunts and normal pulmonary arterial pressure are the most common. These patients are asymptomatic, and the cardiac lesion is usually found during routine physical exam. A VSD is considered an acyanotic congential heart defect. However, if uncorrected, over time the left-to-right shunt can reverse to a right-to-left shunt (Eisenmeger phenomenon) and result in cyanosis. Smaller VSDs are associated with louder murmurs but larger defects may lead to tachypnea and sweating with feeds and failure to thrive. Most small VSDs close on their own while larger VSDs may require surgical intervention if there is associated aortic regurgitation, pulmonary hypertension, pulmonic stenosis or congestive cardiac failure resistant to medications. The murmur of aortic stenosis (A) is characterized by a harsh crescendo-decrescendo systolic murmur, heard loudest at the base of the heart and radiating to the neck. In an atrial septal defect (B), the first heart sound may be normal or split, there may be a midsystolic pulmonary ejection murmur at the upper left sternal border, and in older children, the second hart sound may have fixed splitting. A patent ductus arterosus (C) on auscultation presents as a continuous machine-like murmur.

An 81-year-old man with 10 years of coronary artery disease presents with chest pain and altered mental status. His ECG shows consecutive, large and wide QRS complexes. P waves cannot be appreciated. His pulse is 188 bpm. Which of the following is the most likely diagnosis? Atrial fibrillation Atrial flutter Ventricular fibrillation Ventricular tachycardia

Correct Answer ( D ) Explanation: Ventricular tachycardia is described electrocardiographically as "continuous PVCs", or more specifically, consecutive, fast, large and wide QRS complexes. A wide QRS complex clues one in that the underlying electrical problem is in the ventricles. Ventricular tachycardia can be further defined as monomorphic (QRSs are the same form) versus polymorphic (differing appearance of the QRSs), or sustained (>30 seconds) versus non-sustained (<30 seconds, self-terminates). In this tachydysrhythmia, the ventricles depolarize and contract so fast that cardiac output decreases, especially in the elderly or those with diseased myocardium or coronaries. This perpetuates ventricular ischemia leading to a precipitous decline if the dysrhythmia is not treated. Atrial fibrillation (A) and atrial flutter (B) both have narrow QRS complexes, either irregular or regular respectively. Ventricular fibrilation (C) is identified by a totally erratic appearance of unidentifiable waves. Discrete QRS complexes are absent.

A 68-year-old man is being managed on a multi-drug antihypertensive regimen for essential hypertension. His blood pressure is at goal, but he notes persistent constipation and bilateral pedal edema. Which of the following medications is most likely responsible for these side effects? Enalapril Losartan Metoprolol Verapamil

Correct Answer ( D ) Explanation: Verapamil, along with most other calcium channel blockers, is known to cause pedal edema and constipation, particularly in elderly patients. Other adverse effects of calcium channel blockers include headaches, flushing, and bradycardia, especially with the dihydropyridine calcium channel blockers (such as nifedipine, amlodipine, or nicardipine). Edema development can usually be lessened by adding an ACE inhibitor or angiotensin receptor blocker to the patient's regimen. Calcium channel blockers are generally well tolerated and valuable components of an antihypertensive regimen that work by causing peripheral vasodilation while minimizing the reflex tachycardia common with other vasodilating agents. Hypertension is diagnosed when a patient's systolic blood pressure exceeds 140 mm Hg, or their diastolic blood pressure exceeds 90 mm Hg on two separate occasions. If a work-up for secondary causes of hypertension proves fruitless, a patient is presumed to have essential hypertension and should be managed with diet, exercise, and medications to reach their goal blood pressure (acceptable goals range from 120-140/90-100 depending on patient circumstances). Proper control is vital to reducing complications such as cardiovascular disease, cerebrovascular disease, hypertensive nephropathy, and hypertensive retinopathy. A medication regimen must be tailored for each patient, but will generally include some combination of diuretics, ACE-inhibitors, angiotensin receptor blockers, beta-blockers, calcium channel blockers, or centrally-acting sympatholytics. Enalapril (A), an ACE-inhibitor, is a common initial medication for treating hypertension. However, it is more likely to cause a chronic, dry cough and hyperkalemia rather than pedal edema or constipation. Losartan (B) is an angiotensin receptor blocker. This class of antihypertensive medications has similar utility to an ACE-inhibitor, and it also is likely to contribute to the development of hyperkalemia. It is unlikely to cause the edema and constipation seen with calcium channel blockers. Metoprolol (C), a beta-blocker, is also used for managing hypertension by decreasing the heart rate and cardiac output. Beta-blocker side effects normally include fatigue, lethargy, bronchospasm, and bradycardia, but do not commonly cause constipation or pedal edema.

A 32-year-old man with a history of intravenous drug use presents to the emergency department with complaints of chest pain, shortness of breath, cough and fever. Physical exam reveals a temperature of 102OF, new heart murmur, diffuse petechial rash and subungual hemorrhages. Which of the following is the most appropriate next step in management? Begin anticoagulation therapy Begin empiric antibiotic therapy Obtain three sets of blood cultures Order cardiovascular surgical consult

orrect Answer ( C ) Explanation: Infective endocarditis (IE) affects the endocardial surface of the heart including the valves and mural endocardium. Different types of endocarditis have different pathogens and etiologies. Intravenous drug use is a risk factor for bacterial endocarditis. Diagnosis may be challenging and requires a high level of suspicion, as patients generally have no previous cardiac disease or heart murmurs. Patients often present with nonspecific complaints of fever, chills, night sweats, myalgias, joint pain, anorexia, and weight loss. Clinical manifestations include fever and heart murmurs. Classic signs of IE include petechiae, subungual hemorrhages, tender nodules on the fingertips, and nontender macules on the palms and soles. In the emergent setting, initial goals include stabilizing the patient and making the correct diagnosis. Three sets of blood cultures should be obtained over the first 60-90 minutes and then empiric antibiotic therapy may be administered based on the patient's history and risk factors. The use of anticoagulation therapy (A) is controversial in the treatment of IE and it does not play a role in initial management. One of the primary goals in the diagnosis of IE is to determine the correct diagnosis by identifying the pathogen involved. Administration of empiric antibiotic therapy (B) before obtaining blood cultures could interfere with determining the most appropriate anti-infective agent. Some patients with IE will eventually need surgery and a cardiovascular surgical consult (D) may be appropriate later in the treatment process, but is not part of the initial management of a patient with IE.

hich of the following best classifies this fracture? Salter-Harris Type I Salter-Harris Type II Salter-Harris Type III Salter-Harris Type IV

orrect Answer ( C ) Explanation: The Salter-Harris classification of fractures describe pediatric fractures through the growth plate. These fractures are classified according to the involvement of the physis, metaphysis, or epiphysis. The classification of the injuries is important, because it affects the treatment and prognosis for the patient. The radiograph above demonstrates a Salter-Harris Type 3 fracture. This is defined by a fracture involving the physis (growth plate) and epiphysis. The fracture passes through the hypertrophic layer of the physis and extends to split the epiphysis. Because it crosses the physis and extends into the articular surface of the bone, Salter-Harris Type III fractures put the patient at risk for chronic disability. Rarely does significant deformity occur and the overall prognosis is favorable. Definitive treatment for Type III fractures is surgical. Salter-Harris Type I (A) is a transverse fracture through the hypertrophic zone of the physis. This leads to a radiographic widening of the physis. The growing zone of the physis usually is not injured, and growth disturbance is uncommon. Clinically, the patient usually exhibits point tenderness at the epiphyseal plate, which is suggestive of a type I fracture. Salter-Harris Type II (B) is the most common type of Salter-Harris fracture, and occurs through the physis and metaphysis. The epiphysis is not involved in the injury.These fractures may cause minimal shortening; however, the injuries rarely result in functional limitations and the overall prognosis is good. A Type IV fracture (D) involves all 3 elements of the bone, passing through the epiphysis, physis, and metaphysis. Similar to a type III fracture, a type IV fracture is an intra-articular fracture and can result in chronic disability. Definitive treatment for Type IV fractures is surgical. A Type V fracture is a crush fracture of the epiphyseal plate (opposite of a Type I) and most commonly occurs in the knee and ankle. Functional prognosis is poor because often the blood supply to the physis is interrupted.


Conjuntos de estudio relacionados

Pretransfusion and Compatibility Testing

View Set

Sports Marketing Exam Study Guide

View Set

Dimensions of Causal Attribution

View Set